RR - Cardiology 3

अब Quizwiz के साथ अपने होमवर्क और परीक्षाओं को एस करें!

What is the mechanism of action of adenosine?

AV nodal conduction blockade.

What are indications for emergent dialysis?

Acidosis, electrolyte disturbance, intoxication, volume overload, uremia (AEIOU).

What is the name for an episode of transient monocular blindness caused by acute retinal ischemia?

Amaurosis fugax.

Which beta-blockers have a proven mortality benefit in the treatment of heart failure?

Carvedilol, metoprolol succinate and bisoprolol.

In a patient traveling from Central America, what is a leading cause of myocarditis?

Chagas' disease caused by Trypanosoma cruzi.

What is the name of the rapid quick arterial pulse seen in aortic regurgitation?

Corrigan's pulse.

How long after an acute myocardial infarction does a left ventricular aneurysm occur?

Days to weeks.

What are the components of the San Francisco syncope rule?

History of CHF, hematocrit <30%, abnormal ECG, dyspnea, systolic BP <90—defines high-risk criteria for patients with syncope.

What structural heart disease can cause sudden cardiac death in young people?

Hypertrophic cardiomyopathy.

List some common side effects of IV labetalol?

Hypotension (as with other IV antihypertensives), dizziness, paresthesias (especially of the scalp) and bronchospasm.

Cardiopulmonary Arrest

Improved outcomes: • AED use • Early bystander CPR • Presenting rhythm: v-tach/v-fib • CPR prior to defibrillation • Amiodarone use in shock resistant v-tach/v-fib • Therapeutic hypothermia

Where is the most common anatomic location of an AAA?

Infrarenal.

What is the definitive treatment for third-degree heart block?

Internal pacemaker placement.

Which antihypertensive agent is preferable for a hypertensive emergency caused by a pheochromocytoma?

Intravenous phentolamine is an alpha-blocker which can be used to block the catecholamine effects of a pheochromocytoma.

What effect does the Valsalva maneuver have on the murmur associated with aortic stenosis?

It decreases the murmur.

What electrolyte deficiency is associated with Prinzmetal angina?

Magnesium deficiency.

Mitral valve prolapse is often associated with what conditions?

Marfan's syndrome and Ehlers-Danlos syndrome.

Ventricular Fibrillation

Most common cause: ischemic heart disease • ECG: irregular chaotic pattern without P waves or QRS complexes • Immediate defibrillation

Which class of medications should be given initially for acute pulmonary edema?

Nitrates.

What medication is recommended for the treatment of atrial dysrhythmias associated with hyperthyroidism?

Propranolol.

What medication, if used within the last 24 hours, is a contraindication to administration of nitroglycerin?

Sildenafil.

Which arteries must be evaluated for aneurysm in any patient with a diagnosed AAA?

The femoral (85% association) and popliteal (62% association) arteries.

Which criteria are used in securing a diagnosis of bacterial endocarditis?

The modified Duke criteria.

What is the inherent pacemaker rate of the AV node?

The rate is 40-60 beats/min.

What is the recommended INR for a mechanical mitral valve?

The target INR for a mechanical mitral valve is 3-3.5, whereas a mechanical aortic valve is 2.5-3.

Mitral Valve Prolapse

Thin females • Marfan's, Ehlers-Danlos • Mid-systolic click with late systolic crescendo murmur • ↑ Preload → ↓ murmur; ↓ preload → ↑ murmur • Usually asymptomatic

What are some lifestyle modifications recommended for those with heart failure?

Tobacco cessation, limited alcohol and sodium intake, weight loss and regular exercise.

Which class of migraine abortive medications should be avoided in patients with Prinzmetal's angina?

Triptan medications, which can exacerbate coronary vasospasms, should be avoided in patients with Prinzmetal's angina.

True or false: All patients aged 40 years and older with diabetes mellitus should receive statin therapy?

True.

What dysrhythmia are patients with WPW at risk for if AV nodal blockers are used for antidromic AVRT?

Ventricular fibrillation.

What is the common ventricular rate seen with atrial flutter?

150 bpm, due to the common 2:1 atrioventricular blockade.

What is the most common cause of acute coronary syndrome?

Atherosclerosis

What is the long-term treatment for paroxysmal supraventricular tachycardia (PSVT)?

Calcium channel blockers, beta-blockers and radiofrequency catheter ablation.

A 41-year-old woman presents with fever and retrosternal chest pain of 28 hours duration. A loud friction rub is appreciated over the left sternal border. There is widespread ST-segment elevation seen on her ECG. Which of the following physical findings do you most expect to find in this patient? Chest pain lessens when she sits forward Chest pain worsens during forced expiration Normal breath sounds over the left chest field Tympanic percussion over the left chest field

Chest pain lessens when she sits forward The pericardium is a two layered sac (visceral and parietal pericardia) which surrounds the heart and great vessels. It normally contains 15-50 ml of straw-colored fluid. The pericardium can become inflamed (pericarditis), or become filled with fluid (effusion) or become constrictive (as an end result of inflammation). Acute pericarditis is most commonly caused by an infectious etiology (mainly viral, but possibly bacterial, tuberculous or fungal) or unknown source (idiopathic, 80% of which self-resolve within 1-3 days). It can also arise from neoplastic, autoimmune, radiation or uremic insult. In addition, pericarditis can be associated with acute transmural myocardial infarction or proximal aortic dissection. Pericarditis is marked by pleuritic chest pain that is less with sitting forward and worse with recumbency, effusion and tamponade, and friction rub, a rough sound heard best at the left lateral sternal border with the diaphragm. Viral, bacterial, tuberculous and fungal sources are possible. ECG, echo, chest radiograph, CBC, ESR and serum troponins are the preferred initial diagnostic tests. Treatment includes NSAIDs, corticosteroids and colchicine. Drainage and antibiotics are usually necessary if an infectious etiology is found. The pain of pericarditis is pleuritic; that is, it is worse with deep inspiration, not expiration (B). As effusion is common with pericarditis, an increase in pericardial sac volume can have a mass effect on surrounding tissue, namely the left posterior lung. This can lead to compressive atelectasis, decreased breath sounds (C) and dullness (D) to percussion (Ewart's sign).

Which bile acid sequestrant medication can lower A1C by 0.5% in patients with type 2 diabetes and dyslipidemia?

Colesevelam.

Up to 40% of patients with GCA present with which atypical symptoms?

Cough, choking sensation, neuropathy and extremity claudication.

What are some common side effects of beta blockers?

Depression, fatigue and sexual dysfunction.

List some causes of secondary hypertension?

Diabetes, glomerulonephritis, renal atherosclerosis, Cushing's syndrome, hyperaldosteronism, pheochromocytoma, obstructive sleep apnea, aortic coarctation, meds (OCPs, NSAIDs, steroids, cyclosporine).

Which medications are associated with periodic sinus arrest?

Digitalis, procainamide and quinidine.

What are some anticholinergic side effects?

Dry mouth, blurred vision, constipation, drowsiness, sedation and urinary retention.

Which one of the following is the most common atypical presenting complaint in an elderly patient diagnosed with acute coronary syndrome? Diaphoresis Dizziness Dyspnea Weakness

Dyspnea Atypical features of acute coronary syndrome are present with increasing frequency in sequentially older populations. Elderly patients commonly present with atypical features of acute ischemic heart disease. In fact, atypical features are even more common than chest pain, with 60%-70% of patients older than 85 presenting with an angina equivalent complaint—most commonly dyspnea. Diaphoresis (A), dizziness (B), and weakness (D) are all atypical features of acute coronary syndrome but occur less commonly than dyspnea does.

Torsades de Pointes (TdP)

ECG will show rhythm > 100 beats per minute and frequent variation in the QRS axis and morphology • Most commonly caused by acquired or congenital long QT interval syndrome • Treatment is . Unstable: defibrillation . Stable: intravenous magnesium sulfate and stopping the offending drug

How would you differentiate pericarditis from myocarditis?

Echocardiogram.

An elderly woman presents to the clinic 2 weeks after a lengthy hospital stay. She had repair of a hip fracture which required prolonged bed rest. Her main complaint today is an area of "painful redness" on her leg. Examination reveals a nodular, rope-like, tender superficial vein on the anteromedial proximal thigh. You order a duplex ultrasound which reveals superficial vein thrombus with extension into the deep veins of the thigh. Which of the following is the most appropriate when considering anticoagulation in this patient? Aspirin Enoxaparin Ibuprofen Unfractionated heparin

Enoxaparin Phlebitis is the counterpart to vasculitis, in that phlebitis is inflammation of a vein. Thrombophlebitis is vein inflammation due to thrombus (blood clot). These commonly occur in the superficial veins of the lower extremities, whereas, deep venous thrombosus (DVT) is the term used when a clot lodges in the deep veins. Symptoms include painful, palpable cord-like veins, distal edema and possibly localized erythema. Causes include a predilection to clotting, vein injury, incompetent venous valves (varicosity), Behcet's syndrome and prolonged travel or immobility. Evaluation usually includes Doppler ultrasonography and blood clotting studies. The key to treatment is driven by where the disorder is located. Below-the-knee superficial thrombophlebitis is usually treated with NSAIDs, compressive support and leg elevation. Above-the-knee superficial thrombophlebitis is commonly associated with deep venous thrombus. As such, dislodgement of the deeper clot can cause a life-threatening pulmonary embolus. Therefore, it is important to ascertain whether or not an above-the-knee superficial thrombophlebitis occurs with a deep venous clot (via Duplex ultrasound). If it does, then anticoagulation is recommended as part of the treatment plan. If it isn't, the recommended treatment is less clear. Enoxaparin is a low molecular weight heparin (LMWH) that is used to prevent and treat deep vein thrombosis or pulmonary embolism. It is administered as a subcutaneous injection and does not require aPTT monitoring which makes it ideal for outpatient treatment. Aspirin (A) inhibits platelet function but does does not have anticoagulant properties and is therefore not a recommended treatment for DVT. Ibuprofen (C) has a platelet inhibitory effect but is not an anticoagulant and is therefore not a recommended treatment for DVT. However, NSAIDs are the treatment of choice for superficial thrombophlebitis. Unfractionated heparin (D) is administered intravenously and requires monitoring of the aPTT every 6 hours which is not conducive for outpatient therapy. LMWH preparations offer more reliable anticoagulation, can be used in the outpatient setting, do not require aPTT monitoring and are associated with a lower risk of bleeding complications.

What is the INR range in a patient taking warfarin for atrial fibrillation?

2-3.

What is Levine's sign?

A clenched fist over the chest in patients with ischemic chest pain.

What is an Austin-Flint murmur?

A variant of aortic regurgitation, the Austin-Flint murmur is best heard with the bell at the apex, is more low-pitched and rumbling, and can be presystolic or mid-diastolic.

Prior to electrical cardioversion in a hemodynamically stable patient with atrial fibrillation that has been present for > 48 hrs, what test needs to be done to assess for atrial clot formation?

Transesophageal echocardiogram.

Transient Ischemic Attack

Transient episode of neurological dysfunction without acute infarction • 10% of TIA patients will have a stroke within 90 days • Aspirin + dipyridamole or clopidogrel monotherapy • ABCD2 score: predicts likelihood of subsequent stroke within 2 days

A 22-year-old man with a history of intravenous drug use presents to the Emergency Department with fever, chills, cough, and hemoptysis. His chest X-ray is shown above. Which of the following valves is the most likely involved? Aortic valve Mitral valve Pulmonic valve Tricuspid valve

Tricuspid valve This patient is presenting with signs and symptoms of infectious endocarditis. Risk factors for infectious endocarditis include rheumatic heart disease, congenital or acquired valvular disease, and intravenous drug use. Left-sided endocarditis involves either the aortic or mitral valve. It is more common than right-sided endocarditis. Right-sided endocarditis involves either the pulmonic or tricuspid valve (more common). It is classically seen in intravenous drug users. This patient's chest X-ray demonstrates septic emboli, a known complication of infectious endocarditis. Organisms implicated in right-sided endocarditis include Staphylococcus aureus, Streptococcus pneumoniae, and gram negative bacteria. Presenting symptoms often include fever, cough, hemoptysis, chest pain, and dyspnea. Right-sided endocarditis is frequently misdiagnosed initially as pneumonia. Management includes antibiotics for the suspected organism based on the clinical situation. In an IV drug user, coverage should include methicillin-resistant Staphylococcus aureus and Pseudomonas aeruginosa in addition to the typically implicated organisms. The most appropriate antibiotics for this patient would include cefepime and vancomycin. The aortic valve (A) and mitral valve (B) are unlikely to be involved in patients with intravenous drug use and the chest X-ray above is highly suggestive of septic pulmonary emboli, which are seen in right-sided endocarditis. The pulmonic valve (C) while on the right side of the heart, is uncommonly affected by endocarditis in general and also very atypically involved in patients with intravenous drug use.

The risk of death in unstable angina is four-fold increased if which three biomarkers are elevated?

Troponin I or T, high-sensitivity C-reactive protein and B-type natriuretic peptide.

True or false: Medical providers should counsel their patients to include 40-minute sessions of moderate to high-intensity aerobic activity 3-4 times per week as part of a healthy lifestyle?

True.

What is the greatest risk factor for sudden death from ventricular fibrillation?

Underlying left ventricular dysfunction (LV EF < 30-35%).

Name some thrombophilic disorders or situations?

Use of oral contraceptive pills or hormone replacement therapy, antiphospholipid antibody syndrome, protein C or S deficiency, hyperhomocysteinemia and Factor V Leiden (excess Factor V).

Supraventricular tachycardia is most common in what population?

Young, healthy individuals with no underlying heart disease.

In which peripheral vascular condition would revascularization or bypass be a course of treatment?

In peripheral artery disease revascularization or bypass may be a course of treatment if anticoagulation is not sufficiently reducing ischemia.

Isolated tricuspid regurgitation is most often seen in which group of patients?

Intravenous drug abusers, secondary to tricuspid valve endocarditis.

A 58-year-old man presents with 3 months of headache and diplopia. He also reports that chewing tough foods has been progressively painful. Laboratory testing reveals a normocytic anemia and ESR of 88 mm/hour. A brain CT is normal. Which of the following is the most appropriate initial intervention? Intravenous methylprednisolone Oral methotrexate Temporal artery biopsy Ultrasonography

Intravenous methylprednisolone Giant cell arteritis (GCA), as known as temporal arteritis, is the most common primary vasculitis of the elderly, and predominately affects those aged over 50 years. It is a syndrome of systemic inflammation which mainly affects the branches of the internal and external carotid arteries, but it can affect any vessel in the body. The most common symptoms are a temporal or occipital headache, jaw claudication (mandibular, tooth and tongue pain with chewing tough foods), visual changes (partial or complete blindness, diplopia, visual field cuts or amaourosis fugax) and arthralgias. It is associated with constitutional symptoms, anemia, an enlarged tender nodular erythematous temporal artery, ESR > 50 mm/hour and polymyalgia rheumatica. Temporal artery biopsy confirms the diagnosis. Since unilateral partial or complete blindness occurs in up to 20% of patients with GCA, immediate treatment is necessary when this condition is suspected. High-dose corticosteroids are the standard treatment, and typical options include 40-60 mg per day of oral prednisone, or intravenous methylprednisolone. High-dose therapy is usually continued for 2-4 weeks, and titrated against visual and headache improvement, not serial ESR monitoring. After symptoms lessen, steroids are slowly tapered over 6 months, with complete tapering off in 2 to 3 years. As such, the clinician must monitor for complications of long-term corticosteroid therapy. These include hypertension, diabetes, osteoporosis, steroid myopathy (proximal weakness), fluid retention, bruising, insomnia, restlessness, hypomania and hypercholesterolemia. Adjuvant methotrexate (B) is not routinely recommended for treating GCA as studies of its efficacy are inconclusive. If GCA is suspected, intravenous methylprednisolone should be started even before a temporal biopsy (C). Steroids will not affect the validity of a biopsy result until about 3-4 weeks of treatment, however, a delay in steroids may lead to blindness. There is some evidence of the usefulness of temporal artery ultrasonography (D) for detecting the best area of the artery to biopsy. However, a negative ultrasound does not rule out GCA, and furthermore, some studies have shown that temporal artery ultrasound sometimes cannot differentiate degenerative changes from inflammatory changes.

What is the leading cause of acquired heart disease in kids younger than 5 years of age?

Kawasaki disease.

Which two components of the cardiac cycle are impaired in heart failure with preserved ejection fraction?

Left ventricular filling and relaxation.

What is the name of the non-infectious endocarditis that presents with sterile vegetations on both sides of the involved valve?

Libman-Sacks endocarditis.

Prosthetic Heart Valve Complications

Lifespan: mechanical > biologic • Anticoagulation: mechanical only • Click: mechanical only • Paravalvular leak • Endocarditis

A 62-year-old previously healthy, nonsmoking man presents to your office with questions about his cholesterol. His recent lab results include total cholesterol 245 mg/dL, high- density lipoprotein 38 mg/dL, low-density lipoprotein 165 mg/dL and triglycerides 250 mg/dL. He has an ASCVD score of 9%. Which of the following is the most appropriate initial therapy? Fenofibrate Fish oil Gemfibrozil Lovastatin

Lovastatin The American College of Cardiology and the American Heart Association (ACC/AHA) released a new set of guidelines in November 2013 for treatment of hyperlipidemia with the focus being to reduce the risk of atherosclerotic cardiovascular disease (ASCVD) in adults. Cardiovascular risk is estimated based on age, total cholesterol, high-density lipoprotein (HDL) cholesterol, low-density lipoprotein (LDL) cholesterol, gender, smoking status, systolic blood pressure and the presence or absence of diabetes. Several online calculators are available to determine risk. Four groups have been identified as benefiting from statin therapy: patients with ASCVD, patients with LDL levels greater than or equal to 190 mg/dL, patients aged 40-75 years with diabetes and an LDL level of 70-189 mg/dL, and patients with an LDL level of 70-189 mg/dL and a 10-year ASCVD risk of greater than or equal to 7.5%. The starting point to reduce the risk of ASCVD is lifestyle modifications such as regular physical exercise, heart-healthy diet, smoking cessation and maintaining a healthy weight. First-line treatment for patients meeting criteria for treatment of hypercholesterolemia is statin therapy, including lovastatin. This patient meets criteria for treatment based on his ASCVD score of > 7.5%. Fenofibrate (A) and gemfibrozil (C) are used in the treatment of hypertriglyceridemia. Fish oil (B) is recommended for patients with refractory hypertriglyceridemia and not as first line for hypercholesterolemia.

Which heparin should be avoided in patients with severe renal insufficiency?

Low molecular weight heparin is almost exclusively renally cleared and should be avoided with creatinine clearance < 30 mL/min.

PDE-5 Inhibitors

Mechanism: inhibits PDE-5 → corpus cavernosum smooth muscle relaxation + ↑ blood flow → erection • Headache, dyspepsia ("Hot and heavy, but then headache, heartburn, hypotension") • Nitrates + PDE-5 inhibitor use → refractory hypotension

Venous ulcers are more commonly seen on the medial or lateral malleolus?

Medial.

A woman presents with reduced exercise tolerance and dyspnea on exertion. On exam, you hear the murmur heard in the audio clip above. Which of the following is the correct diagnosis? Aortic regurgitation Aortic stenosis Mitral regurgitation Mitral stenosis

Mitral stenosis The murmur of mitral stenosis includes a loud S1 and an opening snap in early diastole accompanied by a low-pitched, rumbling diastolic apical murmur. Mitral valve stenosis most commonly results from rheumatic heart disease. Other causes include atrial myxoma, congenital abnormalities, and calcific valve degeneration. The murmur of mitral stenosis occurs from mitral valve narrowing leading to increasing pressures across the mitral valve with subsequent left atrial hypertrophy and eventual dilation with left heart failure. The murmur of aortic regurgitation (A) is a blowing diastolic murmur heard best at the sternal border. The murmur of aortic stenosis (B) is a crescendo-decrescendo systolic murmur that radiates to the neck. The murmur of mitral regurgitation (C) is a loud holosystolic murmur heard best at the apex and radiates to the base.

A 73-year-old man with a history of hypertension and COPD presents with the ECG seen above. Which of the following is the correct diagnosis? Atrial fibrillation Atrial flutter Multifocal atrial tachycardia Wandering pacemaker

Multifocal atrial tachycardia Multifocal atrial tachycardia (MAT) is a subset of atrial tachycardia, with more than 2 foci of impulse formation. On the ECG, at least 3 distinctively different P waves with varying P'-R, R-R, and P'-P' intervals are seen. MAT is often associated with pulmonary disease (e.g., COPD) and hypoxemia, either directly from these conditions or as a result of beta-adrenergic agonist or chronic methylxanthine treatment. MAT often resolves when the underlying condition is treated. MAT is easily confused for atrial fibrillation because both rhythms may be irregular and tachycardic, but P waves exist in MAT, not atrial fibrillation. Atrial fibrillation (A) is characterized as an irregularly irregular rhythm where there are multiple atrial foci firing rapidly and continuously, creating a fibrillating baseline. The QRS complexes are occurring irregularly but are usually of normal duration and configuration. The ECG above has P waves, which are not present in atrial fibrillation. Atrial flutter (B) is a fast atrial tachycardia (with an atrial rate upwards of 250-300 b/min) that can be due to a re-entry problem and is characterized by a sawtooth pattern (created by the rapidly firing atrial foci); the QRS complexes are typically regular. An irregular rhythm can occur with variable blocks. Wandering pacemaker (D) is similar to MAT in that there are 3 different P wave morphologies However, the rate in wandering pacemaker is 60-100 beats per minute, slower than MAT.

What is the most common tachydysrhythmia seen in Wolff-Parkinson-White syndrome?

Orthodromic atrioventricular reentrant tachycardia.

Kawasaki Disease

Patient will be a child < 4 years old • With a history of high fever for 5 days • Complaining of conjunctivitis, rash, adenopathy, strawberry tongue, hand/feet edema, fever • Treatment is IVIG + aspirin • Comments: #1 cause of pediatric acquired heart disease, risk for coronary artery aneurysm • Mnemonic: CRASH and burn: Conjunctivitis, Rash, Adenopathy, Strawberry tongue, Hand/feet edema, Fever

Varicose Veins

Patient will be a woman • Complaining of a dull ache in her legs after prolonged standing • PE will show dilated, elongated subcutaneous veins • Diagnosis is made by duplex ultrasound • Treatment is leg elevation and compression stockings

Temporal Arteritis (Giant Cell Arteritis)

Patient will be a woman > 50 y/o • Complaining of monocular visual loss, unilateral headache, jaw claudication • PE will show tender temporal Artery • Labs will show ESR > 50 • Diagnosis is made by temporal artery biopsy • Treatment is high dose steroids ASAP • Comments: Associated polymyalgia rheumatica

Endocarditis

Patient will be complaining of fever, rash, cough and myalgias • PE will show Fever, Roth spots, Osler nodes, Murmur, Janeway lesions, Anemia, Nailbed hemorrhages, Emboli (FROM JANE) • Diagnosis is made by echocardiography and Duke's criteria • Most commonly caused by: . IVDA: S. aureus, tricuspid . Native valve: Streptococci, mitral • Treatment is antibiotics • Comments: GI malignancy: S. bovis

Hyperthyroidism

Patient will be complaining of heat intolerance, palpitations, weight loss, tachycardia, and anxiety • PE will show hyperreflexia • Labs will show low TSH and high free T4 • Most commonly caused by Graves disease (autoimmune against TSH receptor) • Treatment is methimazole or PTU • Comments: Propylthiouracil (PTU) P for pregnant

Dressler's Syndrome

Patient with a history of MI • Complaining of pleuritic chest pain radiating to the back, worse when lying back, improved when leaning forward • PE will show tachycardia and pericardial friction rub • ECG will show PR depression, PR elevation (aVR), diffuse ST segment elevation (concave) • Treatment is colchicine, steroids, NSAIDs (less often)

What is the definitive treatment for sick sinus syndrome?

Permanent pacemaker placement.

Antiplatelet therapy in acute coronary syndrome is focused on treating which pathophysiologic process?

Plaque thrombosis.

How many joules are recommended for defibrillation of pulseless electrical activity?

Pulseless electrical activity (PEA) is not a shockable rhythm; defibrillation is not indicated.

What are the two side effects associated with long term use of flecainide?

QRS widening and PR prolongation.

A 52-year-old man states he took his blood pressure and it was elevated to 180/100 mm Hg. He states that he missed his regular dose of antihypertensive medication because he was traveling for business and returned home today. His blood pressure now is 176/102 mm Hg. The patient is otherwise asymptomatic and has a normal physical exam. What is the most appropriate action? Admission for blood pressure monitoring Obtain an ECG Reduce mean arterial pressure by 25% with an intravenous antihypertensive Resume outpatient medication

Resume outpatient medication This patient has asymptomatic hypertension. Without any symptoms or signs of end organ damage, there is no acute intervention necessary. Patients with elevated blood pressure may have vague symptoms (headache, weakness, fatigue). There is no evidence to support the acute reduction of blood pressure in these patients. The physician may elect to administer a dose of the patient's usual medication or discharge with a plan for resumption of these medications at home. Since the patient in the above clinical scenario has a known diagnosis of hypertension and did not recently take his medications, it is appropriate to have him resume the same medication at the same dose. Patients who have an elevated blood pressure despite taking their antihypertensives will need to be assessed by their primary care physician for alteration of their regimen. Patients do not require admission for blood pressure monitoring (A) when they are asymptomatic. Patients with hypertensive emergency will need admission to the hospital and may require an intensive care unit setting. An ECG (B) is required only if the patient has chest pain or has some other reason suggestive of a dysrhythmia or coronary ischemia. In cases of hypertensive emergency and end-organ damage, the goal is to reduce mean arterial pressure by 25% (C) in the first hour. Normalization of blood pressure should occur over the next 8 to 24 hours. Intravenous medications are preferable for this situation since they have a rapid onset and are titratable.

What is Trousseau's sign of malignancy?

Superficial thrombophlebitis associated with cancer, namely pancreatic carcinoma.

Compare the timing of aortic valve murmurs?

Systolic = stenosis, diastolic = regurgitation.

Low-Output Heart Failure

Systolic dysfunction • More common than high-output heart failure • Causes: ischemic heart disease (MC), HTN, cardiomyopathy, valvular heart disease • ↓ CO, ↑ LVEDP, ↑ systemic O2 extraction ratio • Rx: O2, BiPAP, nitrates, furosemide

What part of the duodenum is most commonly involved in aortoenteric fistulas?

The third and fourth portions.

What life-threatening arrhythmia can QT prolongation progress to?

Torsades de pointes.

Electrocautery during surgery can inhibit pacemaker function. How can the pacemaker be changed into an asynchronous mode during the procedure?

A magnet can safely be placed over the pacemaker during surgery.

A 39-year-old woman presents with palpitations and lightheadedness. Her rhythm strip is shown above. Which of the following treatments is indicated? Adenosine Defibrillation Procainamide Sedation with etomidate following by electrical cardioversion

Adenosine The electrocardiogram reveals a supraventricular tachycardia (SVT). Most cases of SVT result from sustained reentry occurring within the atrioventricular (AV) node, with a minority of cases of SVT resulting from a reentry loop from an ectopic atrial focus. The electrocardiographic hallmarks of SVT are a fast, regular rhythm with a narrow QRS complex. Since the depolarization does not come from the sinoatrial node, P waves do not precede each QRS complex, though P waves may be buried within or seen immediately before or after each QRS complex, known as "retrograde" P waves. Paroxysmal SVT is more common in females than males, with a peak incidence in the late teenage and young adult years. Most patients with SVT do not have underlying heart disease. Common symptoms include palpitations, lightheadedness, and dyspnea. Vagal maneuvers, such as carotid sinus massage and valsalva, are often successful in terminating SVT, especially early in the dysrhythmia course. If vagal maneuvers are ineffective, adenosine is the treatment of choice. The initial dose of adenosine is 6 mg rapid intravenous push, followed by a dose of 12 mg if the first dose is ineffective. Beta-blockers and calcium channel-blockers are alternative agents. Electrical cardioversion is used for refractory SVT or patients who are clinically unstable. Defibrillation (B) is the treatment for pulseless ventricular tachycardia and ventricular fibrillation. Sedation with etomidate following by cardioversion (C) is the treatment for refractory SVT or unstable patients. Procainamide (D) is not a first-line treatment for SVT.

A 65-year-old man presents to the ED for chest pain. You are concerned for acute coronary syndrome and want to administer aspirin, but the patient states that he develops angioedema to aspirin. Which of the following is the most appropriate next step in management? Administer clopidogrel Administer dipyridamole Administer the lower dose, 81 mg of aspirin and observe closely for angioedema Pretreat with corticosteroids and antihistamines and administer full-dose aspirin

Administer clopidogrel A 162-325 mg dose of aspirin taken early in the course of myocardial infarction (MI) has been shown to produce a 23% reduction in 30-day mortality. Patients with an aspirin allergy are at risk for losing this benefit. The use of clopidogrel was shown in the CAPRIE trial to be a sufficient antiplatelet inhibitor when compared to aspirin. Therefore, in patients with true aspirin allergies, clopidogrel should be substituted for aspirin. Dipyridamole (B) is used to inhibit thrombus formation but has not been shown to reduce mortality in acute coronary syndrome. It is commonly used in combination with aspirin for the secondary prevention of stroke and TIA. With a true aspirin allergy, all doses of aspirin (C & D) are contraindicated.

Atrial Fibrillation

Alcohol • Irregularly irregular • No P waves • Narrow QRS unless conduction block or accessory pathway • Unstable: cardioversion • Stable: . Rate control with CCBs, ßBs . < 48 hours duration: cardiovert to sinus rhthym . > 48 hours duration: anticoagulate, echo to r/o thrombus, then cardioversion

Supraventricular Tachycardia

All tachydysrhythmias that arise above the bifurcation of the bundle of His • Characteristics: . Atrial rate 120-200 . Rhythm: regular . Narrow QRS • Causes . Pre-excitation syndromes (WPW) . Mitral disease . Digitalis toxicity . Drugs and toxins . Hyperthyroidism • Treatment: . Vagal maneuvers (Valsalva) . Adenosine (first-line medication), ßBs, CCBs . Unstable: synchronized cardioversion

Patients with bicuspid aortic valve more frequently develop what aortic valvular abnormality?

Aortic stenosis occurs much more frequently than aortic regurgitation.

A 74-year-old man is having a pre-operative ECG performed. What is your interpretation of his ECG? Atrial fibrillation Atrial flutter Normal sinus rhythm Sinus tachycardia

Atrial fibrillation Atrial fibrillation is an irregularly irregular rhythm due to uncoordinated atrial activation and random occurrence of ventricular depolarization. The atria are not contracting but they do discharge electrical impulses to the ventricles. However, no single impulse depolarizes the atria completely, so only an occasional impulse gets through the AV node. It is the most common sustained dysrhythmia in clinical practice. Atrial flutter (B) is a rapid atrial rhythm but due to nodal delay, ventricular response rate is slower. Therefore, atrial flutter always occurs with some sort of AV block so that not all impulses are conducted. The resulting block is often variable (2:1, 3:1, 4:1). P waves have characteristic sawtooth pattern. In normal sinus rhythm (C) and sinus tachycardia (D) the SA node is the pacemaker that causes the atria to depolarize regularly and, thus, the ventricles to depolarize regularly. Therefore, the ECGs for both of these rhythms have P waves and QRS complexes that occur regularly. The difference between these two rhythms is with the rate. The rate of sinus rhythm is 60-99. The rate of sinus tachycardia is >100.

What is the most common sequela of mitral stenosis?

Atrial fibrillation. Increased atrial irritability and hypercoagulability associated with pregnancy combined with increased left atrial pressure, increase the risk of atrial fibrillation and left atrial thrombus formation.

Which tachydysrhythmia is most commonly seen in Wolff-Parkinson-White syndrome?

Atrioventricular reentrant tachycardia (AVRT).

What physical exam findings are specific to high output heart failure?

Bounding pulse with a quick upstroke and a wide pulse pressure.

Besides IV isoproterenol (overdrive pacing), what other IV agent can be used to treat torsades de pointes?

IV magnesium sulfate.

Acute Pericarditis

Idiopathic > viral (Coxsackie) • Pleuritic chest pain radiating to the back • Pain ↓ with leaning forward • Pericardial friction rub • ECG: diffuse STE, PR depression • NSAIDs

Dilated Cardiomyopathy

Causes: idiopathic > viral myocarditis, Chagas disease • Echo: 4 dilated chambers (ventricles > atria) • Systolic pump failure → heart failure • S3, S4 • Peripheral embolization of mural thrombi • Most common dysrhythmia: atrial fibrillation • ßBs, ACEIs, diuretics, digoxin, anticoagulation, AICD

What is the mechanism of action of clopidogrel?

Clopidogrel binds to the platelet adenosine diphosphate (ADP) receptor to irreversibly inhibit activation and aggregation for the life of the platelet.

When considering the diagnosis of a patient with leg pain and swelling, you determine that the pretest probability of deep vein thrombosis is high. Which of the following is the most appropriate test in confirming this diagnosis? Compression ultrasonography D-dimer assay Factor V level Venography

Compression ultrasonography Thrombosis of the deep veins (DVT) not only presents with painful legs but also carries the risk of deadly complication. Risk for DVT include a history of prior DVT, current malignancy, stasis (bed rest, paresis, paralysis, inactivity, recent stroke, air travel > 6 hours), injured endothelium (surgery, trauma, systemic inflammation), and thrombophilia, which is associated with a multitude of disorders. Calf pain, edema, warmth and a palpable cord are common findings. Diagnosis considers several pretest probabilities, both major and minor criteria, as well as compression ultrasonography and the D-dimer level. Compression ultrasonography plus a D-dimer is a safe and efficient approach to diagnose deep vein thrombosis. If the pretest probability of DVT is low, and a D-Dimer is negative, DVT can be excluded. DVT is confirmed if pretest probability of DVT is intermediate or high and the compression ultrasound results are positive. The D-dimer (B) is not recommended if pretest probability of DVT is intermediate or high. Factor V (C) levels are considered in the work-up of thrombophilia after a deep vein thrombus is diagnosed. It is not part of the initial DVT diagnostic approach. Some confusion in diagnosing DVT exists when an ultrasound is negative but pretest probability is high or intermediate. CT or MR venography (D), accompanied by a repeat ultrasound if the D-dimer is ≥ 500 ng/mL, is used when this "gray zone" of DVT is clinically encountered.

A 58-year old man is brought to the ED for chest pain that started 30 minutes prior to arrival while he was jogging in the park. Initially, the patient's cardiac monitor shows sinus tachycardia with a rate of 120 beats per minute. However, while you are interviewing the patient in the resuscitation bay, he suddenly becomes pale, pulseless, and the above rhythm is seen on the cardiac monitor. Which of the following is the definitive next step to manage this rhythm? Chest compressions Defibrillation Epinephrine Synchronized cardioversion

Defibrillation The treatment of ventricular fibrillation depends upon whether the onset was witnessed or unwitnessed. If the cardiac arrest is witnessed (as in this case) and of short duration with an initial rhythm of ventricular fibrillation or tachycardia, the patient should receive immediate defibrillation with 200 joules biphasic (or 360 joules monophasic). Newer biphasic defibrillators are preferred. They have a better first-shock success rate than the older monophasic models have while delivering less electrical current and causing less myocardial cell damage. However, if the downtime is unknown, two minutes of CPR (A) is recommended prior to defibrillation for a shockable rhythm to help "prime" the heart to receive the shock. The main benefit of epinephrine (C) during CPR is derived from its alpha-receptors activity, which produces a direct increase in peripheral vascular resistance. In turn, this leads to a subsequent rise in central aortic blood pressure and an improvement in coronary perfusion pressure during chest compressions. For patients with a shockable rhythm, defibrillation is priority, and while epinephrine may be helpful, it is likely not the definitive step in management.. After the initial dose, epinephrine can be readministered every three to five minutes. Synchronized cardioversion (D) is reserved for those patients with a palpable pulse.

Heart Block: First-Degree

ECG will show PR interval > 200 msecs with regular rhythm • Treatment is not necessary • Comments: most common conduction disturbance and usually represents an incidental finding on an ECG

Wolff-Parkinson-White (WPW) Syndrome

ECG will show short PR interval, delta wave, wide QRS • Most commonly caused by an accessory pathway (bundle of Kent) connects atria to ventricles, bypassing AV node • Definitive treatment is radiofrequency ablation

A 65-year-old woman presents to the emergency department in atrial fibrillation with rapid ventricular response for an unknown duration of time. She was started on heparin and no atrial thrombus was seen on transesophageal echocardiogram. She underwent successful direct current cardioversion and is now back in normal sinus rhythm. What is the minimum period of time she must she remain on anticoagulation after cardioversion? Five days Four weeks One year Three months

Four weeks At least four weeks of anticoagulation is recommended post-cardioversion in patients with atrial fibrillation lasting more than 48 hours or for an unknown duration of time. Although electrical atrial activity is normalized following cardioversion, atrial mechanical stunning and a higher risk of stroke may persist for up to four weeks, and warfarin with a goal INR of 2.0-3.0 must be continued during this time. The risk of thromboembolism after cardioversion can be diminished to less than 1% during the four weeks after cardioversion by the use of a month of therapeutic anticoagulation. For patients who have been in atrial fibrillation for less than 48 hours, anticoagulation is not mandatory because clot formation during that time is unlikely. Five days (A) is not enough time to reduce the risk of cardioversion related thromboembolism. Three months (D) and one year (C) of anticoagulation are not recommended. However, lifelong anticoagulation may be recommended depending on the patient's CHADS2 or CHADSVASc score.

What is the most common cause of peptic ulcer disease in adults?

H. pylori infection.

Which of the following is a cause of torsades de pointes? Drugs that shorten the QT interval Hyperkalemia Hypocalcemia Marijuana use

Hypocalcemia Torsades de pointes may be caused by electrolyte disturbances (hypokalemia, hypomagnesemia, hypocalcemia), antiarrhythmic drugs that prolong the QT interval (procainamide, quinidine, disopyramide), N-acetylprocainamide, droperidol, amiodarone, phenothiazines, haloperidol, tricyclic antidepressants, terfenadine, astemizole, ketoconazole, erythromycin, TMP-SMZ, high-dose methadone, or cocaine. Torsades de pointes is also associated with hereditary long QT interval syndromes. It is a form of ventricular tachycardia manifested by episodes of alternating electrical polarity, with the amplitude of the QRS complex twisting around an isoelectric baseline resembling a spindle. The rhythm usually starts with a PVC and is preceded by widening of the QT interval. Treatment involves intravenous infusion of magnesium sulfate and cardioversion. Drugs that prolong the QT interval and not shorten the QT interval (A) can lead to torsades de pointes. Hypokalemia not hyperkalemia (B) leads to torsades de pointes. Cocaine and methadone not marijuana (D) are a cause of torsades de pointes.

A 67-year-old woman with New York Heart Association class III congestive heart failure presents to clinic for a routine examination. Her chief complaint is worsening peripheral edema. Which of the following abnormalities would you most expect to find during a physical examination? Finger-nail clubbing Jugular venous distension Pancreatic duct congestion Pulsus paradoxus

Jugular venous distension Heart failure takes on many forms, however, the overall pathology is a failure to pump blood forward at a sufficient rate. Etiologies include ischemic heart disease (coronary atherosclerotic disease) and cardiomyopathy. Cardiac failure can be backward, or "congestive", versus forward, or "impaired perfusion". Impaired perfusion symptoms include fatigue, weakness, poor appetite, mental status changes and exercise intolerance. Left-sided congestive failure results in dyspnea, orthopnea, paroxysmal nocturnal dyspnea, pulmonary edema, crackles and pulmonary basilar dullness to percussion. Right-sided failure causes peripheral edema, RUQ discomfort, bloating, ascites, hepatosplenomegaly, hepatojugular reflux, jugular venous distension and increased jugular venous pulsation. Congestive failure is also associated with an S3 heart sound. Finger-nail clubbing (A) is more indicative of pulmonary, instead of cardiac, pathology. Pancreatic duct congestion (C) is not associated with heart failure. Backup of the hepatic vasculature is however. Pulsus paradoxus (D) is a systolic blood pressure drop of ≥ 10 mm Hg during inspiration. It is commonly seen in pericardial tamponade, but not congestive heart failure.

An elderly patient presents to the office complaining of lower extremity skin changes and discomfort that is relieved with elevation. Which of the following would most likely be present on physical exam? Cold lower extremities Diminished pulses Lower extremity edema Palpable cord

Lower extremity edema Lower extremity edema is commonly seen in patients with chronic venous insufficiency. Chronic lower extremity venous disease is the most common vascular disorder and is the result of incompetent venous valves that cause elongation, dilation and tortuosity of veins. The most common symptoms are lower extremity pain or discomfort. Pain associated with venous disease is typically worse when standing, or when seated with the feet dependent for prolonged periods of time and improves with limb elevation and walking. Physical findings include abnormal venous dilation with telangiectasias, reticular veins or varicose veins, edema, inflammation, dermatitis or ulceration. The most common risk factors include advancing age, family history of venous disease, increased body mass index, smoking, a history of lower extremity trauma, prior venous thrombosis and, in women, pregnancy. Cold lower extremities (A) and diminished pulses (B) are more commonly seen in peripheral arterial disease, not venous disease. A palpable cord (D) is more common in superficial thrombophlebitis than venous insufficiency.

A 55-year-old woman presents to the office with progressive dyspnea, paroxysmal dyspnea, orthopnea, and fatigue over the last several months. On auscultation of her heart you hear a low-pitched diastolic rumble best heard in the left lateral decubitus position along with a high-pitched opening snap. Which type of valvular abnormality is associated with these findings? Aortic regurgitaion Aortic stenosis Mitral regurgitation Mitral stenosis

Mitral stenosis The main symptoms of mitral stenosis (MS) are slowly progressive dyspnea and fatigue. Most auscultatory signs of MS are missed if not performed in the left lateral decubitus position. Typically, the first heart sound (S1) is accentuated. A low-pitched diastolic rumble, heard with the bell of the stethoscope over the apex is also present. The high-pitched opening snap (OS) is caused by the abrupt stopping of the domed mitral valve into the left ventricle (also appreciated in most patients midway between the left sternal border and apex). A shorter A2-OS distance indicates a more severe MS. Signs of pulmonary hypertension such as a loud P2 and right ventricular hypertrophy can also be present as MS becomes more severe. Mitral stenosis is defined as the reduced ability of the blood to move from the left atrium to the left ventricle in diastole. It is mostly caused by dysfunction in the mitral valve, which lacks the ability to open its leaflets in diastole. Mitral valve stenosis (MS) is predominantly caused by rheumatic carditis and is more prevalent in female patients. Physical signs of severe aortic regurgitation (A) include a rapid, quick arterial pulse, a wide pulse pressure, an early high-pitched, blowing diastolic murmur heard best over the left sternal border, an S3 gallop, and a low-pitched diastolic murmur at the apex (Austin-Flint murmur).The typical physical signs of severe aortic valve stenosis (B) are diminished carotid pulses (delayed and weak), a sustained apical impulse, a single second heart sound, an S4 gallop, and midsystolic crescendo-decrescendo murmur with late peaking best heard at the base of the heart. In mitral regurgitation (C) patients will display a systolic murmur, most often holosystolic, high-pitched and present at the apex with radiation to the axilla, left scapula, middle back, or left sternal border, depending on the direction of the regurgitant jet.

Tricuspid Stenosis

Most common cause: rheumatic heart disease • almost always occurs with mitral stenosis (MS) • Murmur: . Diastolic murmur along left sternal border . Louder than MS during inspiration • JVP: giant "a" waves

A woman complains of her heart racing, facial flushing and headache after taking an unknown pill in her purse. Which of the following is the most likely medication? Amitriptyline Metformin Metoprolol Nitroglycerin

Nitroglycerin Nitroglycerin can cause reflex tachycardia, flushing and headache. It produces a vasodilator effect on the peripheral veins and arteries with more prominent effects on the veins. Nitroglycerine primarily reduces cardiac oxygen demand by decreasing preload as well as some reduction in afterload. It dilates coronary arteries and improves collateral flow to ischemic regions. Nitroglycerine vasodilates blood vessels by releasing nitric oxide in smooth muscle. This vasodilation can cause hypotension. The elevated heart rate is reflexive in nature; the heart attempts to compensate for the drop in blood pressure. Metformin (B) is a common medication used in the treatment of type two diabetes mellitus. The most common side effects are gastrointestinal, including a metallic taste, mild anorexia, nausea, abdominal discomfort and diarrhea. Metoprolol (C) is a beta blocker with many cardiovascular indications. Depression, fatigue, and sexual dysfunction are commonly cited side effects of beta blockers. Amitriptyline (A) is a tricyclic antidepressant. Tricyclics block muscarinic M1, histamine H1, and alpha-adrenergic receptors, and commonly cause cardiac effects, anticholinergic effects, antihistaminic effects, decreased seizure threshold, sexual dysfunction, diaphoresis, and tremor.

What is the best management of first degree atrioventricular block?

No treatment is necessary.

Is rhythm control superior to rate control in atrial fibrillation?

No, rate- and rhythm-control strategies improve symptoms, but neither has been conclusively shown to improve survival compared to the other.

Cardiogenic Shock

Patient will be complaining of lethargy, confusion, and somnolence • PE will show weak peripheral pulses, rapid heart rate and hypotension < 90 systolic • Most commonly caused by complications of acute MI • Treatment is stabilize with fluids, pressers, O2

Aortic Stenosis

Patient will be older • With a history of diabetes, hypertension • Complaining of dyspnea, chest pain, syncope • PE will show crescendo-decrescendo systolic murmur that radiates to the carotids, paradoxically split S2, S4 gallop • Most commonly caused by degenerative calcification • Treatment is aortic valve replacement • Comments: murmur decreases with Valsalva

Rheumatic Fever

Patient with a history of GAS infection • Complaining of fever, red skin lesions on the trunk and proximal extremities, and small, non-tender lumps located over the joints • PE will show JONES criteria: Joints, Oh, no carditis!, Nodules, Erythema marginatum, Sydenham's chorea • Labs will show anti-streptolysin O, anti-DNase B, positive throat culture, or positive rapid antigen test • Treatment is antibiotics, NSAIDs • Comments: Modified Jones Criteria for a first episode of acute rheumatic fever: need 2 major or 1 major and 2 minor

Mitral Stenosis

Patient with a history of rheumatic heart disease • Complaining of exertional dyspnea, hemoptysis • PE will show loud S1, opening snap, low-pitched, rumbling diastolic apical murmur • Most commonly caused by rheumatic heart disease

Deep Vein Thrombosis (DVT)

Patient with a history of smoking, long distance travel, surgery, oral contraceptives use • Complaining of unilateral leg edema, leg pain, tenderness and warmth • PE will show positive Homan's sign • Diagnosis is made by first ultrasound, Gold Standard: venography • Most commonly caused by stasis, hypercoagulable state, trauma (Virchow's triad) • Treatment is IV heparin and switch to warfarin • Comments: Risk stratification by Well's criteria

What is the most common cause of tricuspid valve stenosis? Bacterial endocarditis Dilation and dissection of aortic root Marfan's syndrome Rheumatic heart disease

Rheumatic heart disease Tricuspid valve stenosis (TS) is mostly caused by rheumatic heart disease and is typically associated with other valvular involvement. TS can also be caused by the carcinoid syndrome and certain connective tissue diseases. Secondary causes of TS (e.g., tumors, thrombi) can also precipitate TS. Clinically, patients may be dyspneic with activity. There is jugular vein distention with a large a wave, indicating atrial contraction against a stiff tricuspid valve. TS is usually treated with percutaneous valvular commissurotomy. Open commissurotomy is performed, or valve replacement if the leaflets and subvalvular structures are not reparable. Bioprosthetic valves or mechanical valves can both be used. If a mechanical valve is chosen, patients start warfarin therapy after tricuspid valve replacement. Patients with mitral regurgitation may have a history of rheumatic fever, endocarditis (A), CAD, or CHF. Endocarditis rarely causes tricuspid valve stenosis. Aortic valve insufficiency is generally acquired through valve infection, dilation and dissection of the aortic root (B), trauma, or long-term degenerative change of the valve, particularly in the setting of hypertension. Primary mitral valve prolapse might be familial and is inherited as an autosomal dominant trait with different rates of penetrance. It is typically found in patients with connective tissue disease, cardiomyopathies, and Marfan's syndrome (C).

Sick Sinus Syndrome (SSS)

SA node dysfunction • Tachycardia-bradycardia syndrome: sinus rate varies from fast to slow and back again • Syncope, palpitations • Definitive rx: pacemaker placement + rate control medication • Untreated SSS → sinus block or sinus arrest

A 71-year-old woman presents with 2 days of dizziness and "almost passing out." Her ECG shows episodes of alternating bradycardia and tachycardia with narrow QRS complexes. Which of the following is the most likely diagnosis? Atrial flutter Digitalis toxicity Sick sinus syndrome Ventricular tachycardia

Sick sinus syndrome Sick sinus syndrome occurs as a result of disease of the sinoatrial (SA) node. It is associated with tachycardia-bradycardia syndrome in which the sinus rate varies from fast to slow and back again. ECG shows an irregular rhythm with pauses in sinus activity. Management depends on the presentation. For rhythms that are too slow, the patient may require a pacemaker. Whereas rhythms that are too fast may be treated with calcium channel blockers or beta-blockers. Atrial flutter (A) is associated with regular tachycardia and saw tooth appearance of P waves. Digitalis toxicity (B) is associated with a high degree AV block. Ventricular tachycardia (D) is not associated with a bradycardia.

Persistent fever and bacteremia are found in a 28-year-old woman with new onset murmur. She does not use intravenous drugs, and does not have any prosthetic heart valves or history of congenital heart disease. However, she has significant dental disease and poor dentition. What is the most common organism responsible for bacterial endocarditis in this patient? Clostridium septicum Escherichia coli Staphylococcus aureus Streptococcus viridans

Streptococcus viridans Endocarditis is infection of the cardiac endothelium or valves or both. Acute cases are associated with normal valves and virulent bacteria, while subacute cases represent smoldering infections of abnormal valves with less virulent bacteria. Risk factors include valve prosthesis, history of rheumatic heart disease or prior endocarditis, mitral valve prolapse or regurgitation, congenital cyanotic heart disease, intravenous drug use, indwelling venous catheter, diabetes, poor dentition, hemodialysis and intracardiac devices. Symptoms include persistent bacteremia or fever and other constitutional symptoms (night sweats, weight loss, fatigue, anorexia). Complications include valve defects, septic emboli leading to stroke, pulmonary embolus or myocardial infarction and immune complex deposition disease such as glomerulonephritis and arthritis. The most common cause of native valve endocarditis in a non-user of intravenous drugs is Streptococcus viridans, especially in those with dental disease, as this organism is normally present in the oral cavity. Clostridium septicum (A) is a rare cause of bacterial endocarditis. However, one should consider this as a source of endocarditis in patients with colonic malignancies, as bowel wall degeneration and hematogenous spread of this bacteria can occur. Gram negative rods, like Escherichia coli (B), only account for 5-6% of all endocarditis cases. Staphylococcus aureus (C) is the most common cause of native valve endocarditis in an IV drug abuser, and is more common than S.viridians in those without dental disease.

What criteria are used to help diagnose infective endocarditis?

The Duke criteria.

Which of the following statements is true regarding the condition associated with the ECG seen above? Pain is often relieved in the supine position The ECG is the most reliable diagnostic tool Thrombolytic therapy is a potential treatment Ventricular dysrhythmias are common

The ECG is the most reliable diagnostic tool The ECG is representative of acute pericarditis. This may be difficult to distinguish from the ECG seen with acute MI, coronary artery spasm, or benign early repolarization. Although there is no single test that is diagnostic for pericarditis, the ECG is the most reliable diagnostic tool. It evolves through stages that occur over time. Over the first few hours to days of the illness, there is diffuse ST segment elevation with concurrent PR segment depression. In the next stages, the ST and PR segments normalize, but the T waves flatten with subsequent deep, symmetrical T wave inversion. The final stage is normalization of the ECG. The pain of pericarditis is often worsened by the supine position (A) and relieved by sitting up. Thrombolytic therapy (C) is contraindicated in pericarditis because its use may precipitate hemorrhagic cardiac tamponade. Ventricular dysrhythmias (D) are rare in pericardial disease. Those patients who do exhibit ventricular dysrhythmias should be suspected of having concomitant myocarditis.

Which tests are commonly obtained under an order of "cardiac enzymes"?

Troponin, creatinine kinase and CK-MB, lactate dehydrogenase and myoglobin.

What effect does nitroglycerin have?

Veno- and arterialdilation, decreasing preload and afterload of the heart.

What is the diagnostic criteria for pre-eclampsia?

Preeclampsia refers to the syndrome of new onset of hypertension and either proteinuria or end-organ dysfunction during pregnancy.

Premature ventricular complexes are often caused by or made worse by what common agents?

Caffeine, alcohol and nicotine.

A 35-year-old woman is seen in the clinic with a chief complaint of frequent palpitations associated with symptoms of pre-syncope. She experiences these episodes two to three times per day. Her resting ECG in the office is normal sinus rhythm with heart rate 65/min. Her physical exam is unremarkable. Which of the following is the most appropriate next step in management? 24-48 hour continuous ambulatory ECG monitor Electrophysiologic testing Implantable loop recorder Post-symptom event recorder

24-48 hour continuous ambulatory ECG monitor 24-48 hour continuous ambulatory ECG monitor, also known as a Holter-monitor, is the most appropriate next step in management. It is one of the most frequently used and cost-effective noninvasive tests used to evaluate cardiac rhythm abnormalities. The clinical utility of the ambulatory ECG recording lies in its ability to continuously examine the patient's cardiac rhythm over an extended period of time during normal routine activity, including any physical and psychological changes. Various rhythm recorders can be used to capture a dysrhythmia. Selection depends on the frequency and duration of symptoms. A Holter monitor is worn for 24-48 hours and is used for evaluation of more frequent symptoms. This patients symptoms occur daily, so a 24-48 hour monitor is likely to capture a possible dysrhythmia during an episode of palpitations or pre-syncope. Several other options to monitor cardiac rhythm abnormalities exist and include event recorders and implantable loop recorders. These monitors are most useful when symptoms are infrequent. A post-symptom event recorder (D) is held to the chest when symptoms occur. The advantage of a post-symptom recorder is the lack of electrode leads, making it more comfortable to carry. Implantable loop recorders (C) allow for continuous ECG monitoring to detect cardiac arrhythmias over months to years. This device is placed subcutaneously, similar to a pacemaker, and can be interrogated non-invasively. A looping event recorder records several seconds of the rhythm prior to the device being triggered and is useful in patients with syncope who may not be able to trigger the recording device. This patient's symptoms are frequent making these devices unnecessary. Electrophysiologic testing (B) is invasive and reserved for those at risk of sudden cardiac death. It is indicated in select, high risk patients with unexplained syncope, particularly those with structural heart disease. Electrophysiologic testing should be considered when noninvasive tests have failed to make a diagnosis. This patient is not likely to have structural heart disease or be at risk for sudden cardiac death based upon her history, physical exam findings and initial testing.

Elective surgical repair should be offered to which of the following patients with asymptomatic abdominal aortic aneurysm? 22-year-old man with Ehlers-Danlos syndrome; aneurysm size 3.5 cm 42-year-old man with hypertension; aneurysm size 3.9 cm with 0.9 cm expansion in 6 months 52-year-old man with positive family history; aneurysm size 4.5 cm 62-year-old man with tobacco use; aneurysm size 4.5 cm with aneurysm growth 0.2 cm in two years

42-year-old man with hypertension; aneurysm size 3.9 cm with 0.9 cm expansion in 6 months Abdominal aortic aneurysm (AAA) is defined as any infrarenal aortic diameter ≥ 3 cm. It occurs in men 5-10x more than women. Risk factors include family history (12-19% of first-degree relatives are affected), male sex, age, atherosclerosis, tobacco use and hypertension. The underlying pathology involves oxidative stress, aortic wall inflammation and proteolytic degradation of elastin and collagen. Abdominal ultrasound screening is recommended in any male aged 65-75 years who has ever smoked. Most AAAs are asymptomatic and found incidentally during other diagnostic testing. However, if found, AAAs must be monitored, as size, and not comorbidity, usually dictates management. Elective surgical correction of an asymptomatic AAA is offered to patients when an AAA grows ≥ 5.5 cm, or grows more than 0.6 to 0.8 cm over 6 months. Therefore, the 42-year-old man with hypertension aneurysm size 3.9 cm with 0.9 cm expansion in 6 months should undergo elective repair. Those with AAAs < 5.5 cm, or growth ≤ 0.6 cm per year (A, C and D) are followed with serial ultrasound monitoring, no matter what the age or underlying comorbidity. The following are guidelines, and need alteration if risk factors are present: < 3 cm (no further testing), 3-4 cm (every 12 months) and 4-4.5 cm (every 6 months). Refer to a vascular specialist if an AAA is > 4.5 cm. It is, however, important to medically maximize these patients in an attempt to decrease the likelihood of rupture and to properly prepare for possible future surgery. This includes tobacco cessation, blood pressure control, LDL lowering to < 70 mg/dL and oral beta-blockade.

Which of the following patients should be classified as having unstable angina? A 51-year-old woman who had chest pain three days ago but now is chest pain free and is found to have a positive troponin with Q waves in leads II, III, and aVF, without ST elevations A 55-year-old woman with a history of hypertension but no prior cardiac disease who complains of one episode of chest pressure that began while pushing her grocery cart and lasted 30 minutes A 65-year-old man with a known history of coronary artery disease who gets chest pain and shortness of breath every time he climbs the steps to his bedroom A 71-year-old man who underwent a coronary catheterization one month ago for early morning chest pain that showed minimal coronary artery disease with no fixed lesions presents with recurrent early morning chest pain that is relieved by nitroglycerin

A 55-year-old woman with a history of hypertension but no prior cardiac disease who complains of one episode of chest pressure that began while pushing her grocery cart and lasted 30 minutes Acute coronary syndrome is a spectrum of entities that includes asymptomatic coronary artery disease, stable angina, unstable angina, acute myocardial infarction, and sudden cardiac death. Unstable angina is broadly defined as angina new in onset that occurs at rest or with minimal exertion, or a worsening change in a previously diagnosed stable angina. Unstable angina should be considered a possible harbinger of acute myocardial infarction. A myocardial infarction (A) is acute, evolving, or recent and, unlike anginal syndromes, involves myocardial cell death and necrosis. These patients exhibit a rise and gradual fall in cardiac biomarkers associated with at least one of the following: ischemic symptoms; development of pathologic Q waves on the ECG; ECG changes indicative of ischemia; or coronary artery intervention. Stable angina (C) is transient, episodic chest pain or discomfort, predictable and reproducible with familiar symptoms that occur from a characteristic stimulus. Symptoms improve with rest or nitroglycerin within a few minutes. Variant angina (D), also known as Prinzmetal's angina, is due to coronary artery spasm. This condition can truly be diagnosed only after visualization of the coronary arteries reveal no or minimal fixed lesions. Variant angina occurs at rest and is usually relieved by nitroglycerin or exercise. It can be associated with ST elevations on the ECG that cannot be differentiated from a STEMI.

According to JNC-8, patients older than 60 years with hypertension and no other medical history, should be treated to which of the following blood pressure goals? < 120/80 mm Hg < 130/85 mm Hg < 140/85 mm Hg < 150/90 mm Hg

< 150/90 mm Hg Hypertension (HTN) is one of the most common diseases encountered in primary care, affecting nearly 30% of the US population. There are two main types, essential (95% of all cases) and secondary. Essential HTN begins between ages 25 and 55 years, is associated with a genetic predisposition and positive family history, and is felt to be due to repeated renal microvascular injury and an underlying hyperactive sympathetic tone. Secondary HTN is considered in those <20 or >50 years of age, or in cases of acute-onset, severe or refractory HTN. JNC-7 defines stage 1 HTN as SBP ≥ 140 mmHg and DBP ≥ 90 mmHg. In comparison, JNC-8 recommends treating hypertensive adults > 60 years old to a goal of < 150/90 mm Hg, and adults 30-59 years old to a goal of DBP < 90 mm Hg. There is insufficient evidence for a systolic recommendation in those < 60 years old and for a diastolic recommendation in those < 30 years old. As such, JNC-8 recommends for hypertensive patients < 60 years old a goal of < 140/90 mm Hg. < 120/80 mm Hg (A), < 130/85 mm Hg (B), and < 140/85 mm Hg (C) are incorrect goals in managing hypertension in patients > 60 years old.

What does the highest (worst) TIMI score of 7 indicate?

A 41% chance that in the next 14 days the patient will die, have a new or recurrent MI or have severe recurring ischemia which requires revascularization.

A 28-year-old south Asian immigrant who is in her second trimester of her first pregnancy presents to the emergency department complaining of worsening dyspnea, orthopnea and lower extremity edema. She has never experienced anything like this before. She has no past medical history; however, she admits to frequent sore throats and ear infections as a child. Which of the following is most likely to be heard on auscultatory exam? A diastolic decrescendo murmur heard at the left lower sternal border A diastolic low-pitched decrescendo murmur best heard at the cardiac apex A holosystolic murmur heard best at the cardiac apex A systolic crescendo-decrescendo murmur best heard at the right upper sternal border

A diastolic low-pitched decrescendo murmur best heard at the cardiac apex A diastolic low-pitched decrescendo murmur best heard at the cardiac apex would be the most likely auscultatory finding on exam. This woman likely has mitral stenosis secondary to rheumatic heart disease. Mitral stenosis encountered in women of childbearing age is nearly always rheumatic in origin. Maternal and perinatal complications during pregnancy in women with mitral stenosis reflect the unfavorable interaction between the normal cardiovascular changes of pregnancy and the stenotic mitral valve. Pregnancy is a high flow state and blood volume, cardiac output, and heart rate are increased. In pregnant patients with mitral stenosis, this increases the pressure across the mitral valve and can lead to pulmonary edema and other signs and symptoms of heart failure. Although mitral stenosis is relatively uncommon in developed countries, it is a common condition in pregnant women with heart disease in areas where rheumatic heart disease is prevalent. Although rheumatic heart disease may affect other valves, including aortic and tricuspid, the mitral valve is the predominant lesion and repeated attacks of rheumatic fever over time causes mitral valve stenosis. A systolic crescendo-decrescendo murmur best heard at the right upper sternal border (D) is characteristic of aortic stenosis. A diastolic decrescendo murmur heard at the left lower sternal border (A) classically describes aortic regurgitation. A holosystolic murmur heard best at the cardiac apex (C) is associated with mitral regurgitation.

What ethnic group in the US has the highest incidence of Kawasaki disease?

Americans of Asian and Pacific island descent.

A 35-year-old woman is being managed for multiple episodes of chest pain that have been awakening her from sleep on several early mornings for the past 2 months. An electrocardiogram after one episode showed ST elevation. Coronary angiography did not reveal any stenotic lesions. She has no cardiac risk factors and is otherwise healthy. Which of the following daily medications is most likely to provide relief from her condition? Adenosine Amlodipine Aspirin Propranolol

Amlodipine Amlodipine is the most appropriate selection to use as daily prophylaxis against the anginal pain caused by Prinzmetal (variant) angina. Amlodipine, as well as other long-acting calcium channel blockers and nitrates, is effective at preventing the coronary vasospasm responsible for the chest pain seen in Prinzmetal's angina. This syndrome most commonly occurs in women under 50 years of age, with a classic presentation being angina in the early morning that awakens a patient from sleep. Of note, Prinzmetal's angina is not brought on by typical factors responsible for aggravating the angina of atherosclerosis, and patients may have no coronary risk factors. The work-up of any chest pain of suspected cardiac origin would include an electrocardiogram which, in this case, would show ST-segment elevation in the distribution of the vasoconstricted coronary artery. Coronary angiography will also be helpful to rule out a stenotic lesion in need of intervention. If Prinzmetal angina without a complicated stenotic lesion is diagnosed, patients should be counseled to avoid substances that contribute to vasoconstriction. Two important contributors are cigarette smoking and the use of cocaine. Treatment should consist of daily calcium channel blockers or nitrates to prevent vasoconstriction, and break-through nitrates as needed for acute episodes. Adenosine (A) is an antiarrhythmic medication used most commonly in the management of supraventricular tachycardia. It is not recommended for Prinzmetal's angina, and occasionally even causes chest pain as an adverse effect. Aspirin (C), though commonly beneficial in cardiovascular risk reduction, does not have a role in managing the episodes of vasoconstriction seen in Prinzmetal's angina. Propranolol (D), a beta-blocker, does not have a role in managing Prinzmetal's angina and has been known to contribute to coronary vasospasm. It is generally only considered in patients with Prinzmetal's angina whose symptoms are related to a stenotic lesion.

A 30-year-old man with ankylosing spondylitis presents for a routine physical. This is the first encounter that you appreciate a murmur during cardiac auscultation. You appreciate a diastolic, high pitched, blowing murmur while listening with the diaphragm in the left sternal border. There is no palpable thrill. Which of the following is the most likely diagnosis? Aortic regurgitation Aortic stenosis Mitral regurgitation Tricuspid stenosis

Aortic regurgitation The murmur of aortic regurgitation is best heard with the diaphragm in the left sternal border. It can be accentuated when a patient sits and leans forward. This is a diastolic murmur, as compared to the systolic stenotic murmur of the same valve. It is often high pitched, blowing and decrescendo in nature. A widened pulse pressure is seen due to incompetence of this valve as it allows previously "pumped-out" blood to return back into the left ventricle. Common causes include rheumatic heart disease, trauma and endocarditis, and it can also be associated with Marfan's syndrome, syphilis and ankylosing spondylitis. Aortic stenosis (B) is a systolic murmur. Mitral regurgitation (C) is a pansystolic murmur best heard at the apex. Tricuspid stenosis (D) is a diastolic rumble heard over the left sternal border at the level of the fourth intercostal space. A right ventricular thrill is commonly palpated.

A 67-year-old man with a history of hypertension and remote abdominal aortic aneurysm repair presents to the emergency department after four episodes of gross hematemesis. He had an episode of emesis with some blood last week but dismissed it as a "stomach bug". He denies any history of alcohol abuse or liver disease. His vital signs on arrival are T 38.5°C, HR 111, BP 100/80, RR 27. His stool is guaiac positive. Which of the following is the most likely cause this patient's symptoms? Aortoenteric fistula Esophageal varices Mallory-Weiss tear Peptic ulcer disease

Aortoenteric fistula An aortoenteric fistula is an abnormal communication between the aorta and the gastrointestinal tract. The most common risk factors for aortoenteric fistulas are abdominal aortic aneurysm and a history of aortic surgery. Gastrointestinal bleeding, including hematemesis, hematochezia, and melena, is often the presenting symptom. The classic triad of gastrointestinal bleeding, abdominal pain, and a palpable mass is rarely present and a history of aortic aneurysm is rarely known at the time of presentation. Fever and sepsis may be present due to seeding of the blood with gastrointestinal flora. Management of aortoenteric fistulas is emergent surgical repair. Esophageal varices (B) are typically a complication of liver failure and portal hypertension. While they are a common cause of upper gastrointestinal hemorrhage, in a patient without an alcohol abuse history or a documented history of liver dysfunction, there is no reason to suspect that this is the cause of his bleeding. Mallory-Weiss tears (C) are linear mucosal tears found in the distal esophagus or proximal stomach due to repetitive retching. A history of gross hematemesis without prior episodes of vomiting is not consistent with a Mallory-Weiss tear. Peptic ulcer disease (D) is a common cause of upper gastrointestinal bleeding. Patients typically do not present with fever and aortoenteric fistula should be suspected in patients with signs of an abdominal aortic aneurysm on examination.

Approximately what percent of patients with an acute MI who present to the ED are discharged from the ED without a diagnosis?

Approximately 2%-4%.

Which of the following is a cause of high output heart failure? Aortic stenosis Arteriovenous fistula Chronic hypertension Dilated cardiomyopathy

Arteriovenous fistula Most patients have heart failure as a result of systolic or diastolic dysfunction with decreased cardiac output and increased systemic vascular resistance. In contrast, high output heart failure is characterized by a hyperdynamic state with increased cardiac output, low systemic vascular resistance, increased oxygen consumption as a result of increased metabolic demand, and low arterial-venous oxygen difference. The hyperdynamic state can result from increased preload such as from mineralocorticoid excess, or decreased systemic vascular resistance due to arteriovenous fistulas (either congenital such as hereditary hemorrhagic telangiectasia or acquired such as AV fistula for hemodialysis), cirrhosis, pregnancy, severe anemia, beriberi or thyrotoxicosis. In most cases, these conditions alone are not the sole cause of heart failure but rather induce heart failure in patients who have an underlying cardiac problem. Low output cardiac failure, characterized by decreased cardiac output, high filling pressures and increased systemic vascular resistance, can occur in the setting of aortic stenosis (A), chronic hypertension (C), or dilated cardiomyopathy (D).

Which of the following treatments has a proven mortality benefit in ST-elevation myocardial infarction? Aspirin Morphine Nitroglycerin Oxygen

Aspirin ST elevation myocardial infarction (STEMI) occurs when there is thrombus formation causing a blockage of a coronary vessel. Common symptoms include gradual onset of chest tightness or pressure that radiates to the arms or neck. The classic teaching is to give morphine, nitroglycerin, oxygen, and aspirin for acute coronary syndrome. However, only aspirin has shown a mortality benefit. In recent prospective studies morphine (B) has been associated with a higher mortality rate but no causative relationship has been established. Although nitroglycerin (C) can help reduce ischemic pain through coronary vasodilation it does not reverse the underlying cause. It is not associated with a mortality benefit. Supplemental oxygen (D) is not associated with a mortality benefit and may even cause harm in non-hypoxic patients.

Which of the following is an indication for permanent pacemaker placement? Asymptomatic Mobitz type I second-degree AV block Asymptomatic Mobitz type II second degree heart block Asymptomatic sinus bradycardia with heart rate of 40/min Asymptomatic three second sinus pauses

Asymptomatic Mobitz type II second degree heart block Asymptomatic Mobitz type II second-degree AV block is an indication for pacemaker placement. This block has a high risk of progressing to complete heart block and should be treated with pacemaker placement, regardless of symptoms. In general, the long term treatment for symptomatic sinus bradycardia or heart block without reversible cause is a permanent pacemaker. These devices are usually placed in the left pectoral area with leads inserted through a vein into the heart. Two general factors guide the decision to place a permanent pacemaker: the association of symptoms with a brady-dysrhythmia and the potential for progression of the rhythm disturbance. Progression is largely dependent on the anatomical location of the conduction abnormality. The location of an AV conduction abnormality, within the AV node or below the AV node in the His-Purkinje system is an important determinant of both the probability and progression rate of conduction system disease. Disease below the AV node, in the His-Purkinje system, is generally considered to be less stable. The most common indications for pacemaker implantation are sinus node dysfunction followed by AV block. Sinus bradycardia in which symptoms such as dizziness, lightheadedness, syncope, fatigue, or poor exercise tolerance are present should be treated. Acquired AV block is the second most common indication for permanent pacemaker placement. Complete, or third-degree AV block, advanced second-degree AV block, symptomatic Mobitz I or Mobitz II second-degree AV block are all indications for pacemaker placement. Asymptomatic sinus bradycardia with heart rate of 40/min (C), asymptomatic Mobitz type I second-degree AV block (A) and asymptomatic three second sinus pauses (D) are not indications for a pacemaker in the absence of symptoms.

A 59-year-old man presents complaining of a severe headache for three days that came on gradually, and is diffuse in nature. His blood pressure is 205/105 mm Hg. All other vital signs are normal. His physical exam and basic laboratory workup are negative. His electrocardiogram is shown above. Which of the following antihypertensives is contraindicated in this patient? Atenolol Enalapril Hydralazine Hydrochlorothiazide

Atenolol Wolff-Parkinson-White syndrome (WPW) is characterized by an accessory conduction pathway that bypasses the AV node, resulting in a shortened PR interval and upsloping QRS wave, known as a delta wave. If conduction through the AV node is slowed or disrupted, patients are at risk for an accelerated tachyarrhythmia and sudden cardiac death. There are two types of WPW, each requiring different treatments. Orthodromic WPW is clinically indistinguishable from AVNRT and can be treated in a similar fashion. Antidromic WPW is more dangerous as it can lead to tachyarrhythmias and death as above. Therefore, any medication that slows AV nodal conduction is contraindicated in antidromic WPW. These can be remembered by the mnemonic "ABCD"-- adenosine, beta-blockers, calcium channel blockers, and digoxin. Atenolol is a beta blocker and is therefore contraindicated. Enalapril (B), an angiotensin-converting enzyme inhibitor, hydralazine (C), an arteriolar smooth muscle relaxant, and hydrochlorothiazide (D), a thiazide diuretic, do not interfere with AV nodal conduction and are therefore safe for use in patients with Wolff-Parkinson-White syndrome.

An elderly patient is admitted to the hospital for multiple episodes of syncope. Telemetry reveals periods of sinus bradycardia in the 30s followed by a tachydysrhythmia with heart rates that fluctuate in the 100 to 160 range. During several of the bradycardia episodes the patient becomes hypotensive and symptomatic with shortness of breath, lightheadedness, and dizziness. Which of the following is the most likely underlying tachydysrhythmia associated with this patient's most likely diagnosis? Atrial fibrillation Atrial flutter Multifocal atrial tachycardia Sinus tachycardia

Atrial fibrillation Atrial fibrillation is the most common tachydysrhythmia associated with sick sinus syndrome. Sick sinus syndrome is defined by electrocardiographic criteria because clinical signs and symptoms, if present, are highly variable. The electrocardiographic characteristics of the sick sinus syndrome include frequent periods of inappropriate, and often severe bradycardia, sinus pauses, arrest, and sinoatrial exit block, often without appropriate atrial and junctional escape rhythms. The failure of escape pacemakers may lead to symptoms including syncope. Alternating bradycardia and an atrial tachydysrhythmia is found in over 50 % of cases. Atrial fibrillation is most common, but atrial flutter and paroxysmal supraventricular tachycardias may also occur. The most common cause of sick sinus syndrome is the replacement of sinus node tissue by fibrous tissue. Symptoms of lightheadedness, presyncope, or syncope are often the reason that the patient seeks medical attention. Other manifestations include increasing dyspnea on exertion, worsening angina, and, in patients with alternating bradycardia and tachycardia, palpitations and other symptoms associated with a rapid heart rate. Atrial flutter (B), sinus tachycardia (D) and paroxysmal supraventricular tachycardia are also associated with sick sinus syndrome; however they are not as common as atrial fibrillation. Atrial flutter is a common dysrhythmia that is associated with similar conditions as atrial fibrillation and are sometimes co-existent. Sinus tachycardia is also associated with this sick sinus syndrome. However, the most common causes of sinus tachycardia are the normal response to exercise and conditions such as fright, anger, or stress. Multifocal atrial tachycardia (C) is not generally associated with sick sinus syndrome. It is defined as a heart rate over 100/min involving at least three distinct P-wave morphologies and is most commonly seen in those with significant lung disease.

Approximately one-third of patients with Wolff-Parkinson-White develop what tachydysrhythmia?

Atrial fibrillation, and if atrial fibrillation conducts rapidly down the bypass pathway, it can degenerate into ventricular fibrillation.

A patient presents with palpitations and dyspnea. She is placed on a cardiac monitor as seen above. Which of the following is the most likely diagnosis? Atrial fibrillation Atrial flutter Ventricular fibrillation Ventricular tachycardia

Atrial flutter Atrial flutter is a dysrhythmia characterized by an atrial rate of 250-300 bpm. The most common site of pathology is a reentrant circuit in the right atrium about the tricuspid valve annulus. Etiologies include hypertensive heart disease, ischemic heart disease, rheumatic heart disease and cardiomyopathy. The resulting tachycardia typically causes decreased cardiac output with symptoms of palpitations, dyspnea, presyncope and fatigue. It is diagnosed electrocardiographically by regularly shaped and spaced P waves which have the appearance of a sawtooth pattern. These P waves will intermittently be followed by narrow QRS complexes. Management includes electrical or pharmaceutical cardioversion and ventricular rate control (calcium channel blockers and beta-blockers). Atrial fibrillation (A) is disorganized electrical activity and defined by an irregular rhythm with the absence of P waves. Ventricular fibrillation (C) produces a 'wavy line' ECG, of which no discernable rate, P wave or QRS complex is seen. This dyshythmia is associated with pulselessness. Ventricular tachycardia (D) is a wide complex tachycardia.

A 60-year-old man presents with nighttime dyspnea. His medical history is significant for chronic hypertension. A recent echocardiogram showed an increase in left ventricular chamber volume but normal ventricular wall thickness. Based on this finding alone, which of the following medications is the most appropriate treatment for this patient's dyspnea? Bisoprolol Propranolol Salmeterol Timolol

Bisoprolol Dilated cardiomyopathy (DCM) is a complication of chronic hypertension and coronary artery disease. Patients with cardiomyopathy typically suffer from symptoms of cardiac failure, especially dyspnea and edema. If cardiac failure is suspected, the initial evaluation for underlying cardiomyopathy includes an echocardiogram. In DCM, the echocardiogram usually demonstrates an enlarged ventricular chamber with normal or decreased wall thickness. Ejection fraction, a marker of systolic function, will also typically be normal or decreased. Treatment of DCM follows the American College of Cardiology and the American Heart Association's guidelines for the treatment of heart failure. In addition to lifestyle modifications, pharmacotherapy includes ACE-inhibitors or ARBs, loop diuretics and beta-blockers. Bisoprolol is a selective beta-1 adrenergic receptor blocker. Bisoprolol, carvedilol, and metoprolol, but not propranolol (B), are the only beta-blockers with proven benefit in heart failure management. Salmeterol (C) is a long-acting beta-2-agonist used in treating bronchospasm and COPD, not heart failure. Although this medication could be used for treating dyspnea in a patient with pulmonary disease, there is no information about this patient's pulmonary status given in the question stem above. Timolol (D) is a beta-blocker used in the treatment of glaucoma, not heart failure.

A patient is being evaluated for palpitations in the emergency department. An ECG reveals a short PR interval, widened QRS and slurred upstroke of the QRS complex. Which of the following is the name of bypass pathway associated with this condition? Bachmann's bundle Bundle of His Bundle of Kent Left bundle branch

Bundle of Kent The bundle of Kent is the bypass pathway associated with Wolff-Parkinson-White syndrome, which is the most likely diagnosis in this patient. Wolff-Parkinson-White syndrome is a pattern of pre-excitation on ECG accompanied by a symptomatic tachycardia. It results from an accessory pathway that directly connects the atria and ventricles and bypasses the AV node. The bypass pathways are accessory connections called Kent bundles between the atrium and ventricle. If a bypass pathway conducts antegrade, a pre-excitation pattern may be seen on the ECG. This includes a short PR interval because the bypass pathway conducts rapidly and faster than the AV node; a delta wave, which is a slurred initial segment of the QRS complex resulting from slow conduction through ventricular tissue instead of the His-Purkinje system; and a widened QRS that consists of fusion between early ventricular activation from pre-excitation with the later ventricular activation resulting from transmission through the AV node. Bachmann's bundle (A) is a branch in the atrial conduction system, which facilitates right to left atrial conduction. The bundle of His (B) is specialized conduction tissue that transmits the electrical impulses from the AV node to the point of the apex of the fascicular branches via the left and right bundle branches. The left bundle branch (D) and right bundle branch then lead to the Purkinje fibers, which provide electrical conduction to the ventricles, causing the cardiac muscle of the ventricles to contract at a paced interval.

A 63-year-old man with a history of stage 3 chronic renal insufficiency presents feeling weak. Blood testing demonstrates a potassium of 6.7. An electrocardiogram is obtained as seen above. What combination of medications is appropriate for the treatment of this patient? Calcium gluconate, albuterol, regular insulin and glucose, furosemide Calcium gluconate, penicillin, warfarin Digoxin, metoprolol, furosemide Regular insulin and glucose, aldactone

Calcium gluconate, albuterol, regular insulin and glucose, furosemide This patient has hyperkalemia (normal potassium range 3.5-5.0), and electrocardiographic changes in the form of peaked T waves. Hyperkalemia can present with nonspecific symptoms ranging from weakness, fatigue, cramps up to cardiac dysrhythmias and paralysis. It should be suspected in patients with deficient potassium excretion, including renal failure and certain medications. The progression of electrocardiographic changes due to hyperkalemia is demonstrated begins with peaked T waves, then prolongation of the PR interval until there is loss of the P wave, followed by a widening of the QRS into a sine wave pattern. Potassium elevates the cardiac resting potential, making cardiac myocyte depolarization more likely. Calcium increases the cardiac threshold potential, thereby restoring the gradient between cardiac resting and threshold potentials. Albuterol and insulin acutely drive potassium into cells. Glucose is administered with insulin to prevent hypoglycemia. Furosemide eliminates potassium from the bloodstream via the kidneys. Neither penicillin nor warfarin (B) will decrease the potassium level. There is no indication for digoxin (C) usage in hyperkalemia and in certain situations it may increase potassium. There is no benefit to beta-blockade with metoprolol in hyperkalemia. Aldactone (D) is a potassium sparing diuretic and won't remove potassium from the body.

An 83-year-old is being evaluated in the emergency department after an episode of syncope. The woman was preparing dinner when she felt her heart start to race. The next thing she remembers is waking up on the floor. She experienced a similar episode about three weeks ago. She has never had anything like this before. Her past medical history is remarkable for hypertension, hyperlipidemia and hypothyroidism. Her medications include lisinopril, atorvastatin and levothyroxine. On physical exam her blood pressure is 142/83, heart rate 76/min, and respiration rate 13/min. Cardiac auscultation reveals no murmur. The remainder of her physical exam is normal. Electrocardiogram reveals normal sinus rhythm with left axis deviation. No cardiac rhythm abnormalities are detected. What is the most likely etiology of this patient's syncope? Aortic stenosis Cardiac dysrhythmia Orthostatic hypotension Vasovagal

Cardiac dysrhythmia Cardiac dysrhythmia is the most likely cause of this woman's syncope. Cardiac dysrhythmias are a common cause of syncope in the elderly population. It is characterized by a brief or absent prodrome and palpitations immediately preceding the event. Several episodes over a short period of time in someone with no history of syncope suggest a dysrhythmia. Given this patient's short prodrome, palpitations and history of a previous similar event makes a cardiac dysrhythmia the most likely etiology. Aortic stenosis (A) is unlikely the cause of her syncope. Aortic stenosis is associated with a crescendo-decrescendo systolic ejection murmur. Syncope related to aortic stenosis typically occurs during exertion and is associated with very severe disease. This patient's syncopal episode occurred while stationary. Additionally, she has no systemic symptoms of aortic stenosis. Vasovagal (D) is the most common cause of syncope in the general population. It is usually triggered by provoking factors such a blood draw or an intense emotion. Prodromal symptoms include feeling warm, sweating, nausea, and pallor. This woman does not report any of these symptoms. Orthostatic hypotension (C) causes syncope upon assuming an upright position from supine or sitting. It is often caused by hypovolemia, medications or autonomic nervous system disorders. This woman was standing while preparing dinner making orthostatic hypotension unlikely.

A 37-year-old woman with a history of Wolff-Parkinson-White presents to the emergency department with shortness of breath and lightheadedness. Her vital signs on arrival are T 36.9°C, HR 160, BP 80/50, RR 27. Her ECG reveals a narrow complex regular tachycardia. Which of the following is the most appropriate next step in the management of this patient? Adenosine Cardioversion Fluid bolus Procainamide

Cardioversion Wolff-Parkinson-White (WPW) syndrome is a ventricular preexcitation syndrome that involves an anatomic accessory pathway that bypasses the AV node and forms a direct electrical connection between the atria and ventricle, placing patients at high risk of supraventricular tachydysrhythmias. Tachydysrhythmias in WPW should be treated according to the patient's clinical stability, with hemodynamically unstable patients receiving immediate electrical cardioversion. The first intervention in orthodromic AVRT in a hemodynamically stable patient is a vagal maneuver. If this fails, intravenous adenosine can be used, followed by AV blocking agents such as beta-blockers or calcium channel blockers. Definitive treatment of WPW syndrome is ablation of the accessory pathway. Adenosine (A) is the first line pharmacologic intervention in hemodynamically stable orthodromic AVRT in whom vagal maneuvers have failed. It is not appropriate in a symptomatic, hemodynamically unstable patient. Administering a fluid bolus (C) is appropriate treatment of hypotension in a patient with hypovolemic or distributive shock but would not be appropriate for a patient that is hypotensive due to a dysrhythmia. Procainamide (D) is the treatment of choice in hemodynamically stable antidromic AVRT and is third line for hemodynamically stable orthodromic AVRT. It should not be used in hemodynamically unstable patients.

Which of the following agents is first line for rate control in atrial fibrillation with rapid ventricular response in the setting of compensated systolic heart failure? Amiodarone Carvedilol Digoxin Diltiazem

Carvedilol The beta blocker, carvedilol, is a first line agent in rate control of atrial fibrillation with rapid ventricular response in the setting of compensated systolic heart failure. Other first line beta blockers include extended release metoprolol succinate and bisoprolol. Although beta blockers do not appear to improve mortality in the setting of heart failure and atrial fibrillation, the rationale behind using them is that there is no evidence of harm with their use. Additionally, the alternatives to beta blockers have significant limitations. Alternatives include calcium channel blockers which have been associated with a greater mortality, digoxin which has a lesser efficacy and amiodarone which has more side effects. Use of beta blockers in patients with heart failure due to systolic dysfunction in the absence of atrial fibrillation, have been shown to reduce hospitalizations and improve survival. So when choosing a rate control agent it is advised to use one with a mortality benefit for concomitant systolic heart failure. Non-dihydropyridine calcium channel blockers such as verapamil and diltiazem (D) are inappropriate as these agents should be avoided in those with a reduced left ventricular systolic function. Digoxin (C) may be considered as second line in patients who cannot receive a beta blocker for rate control. In the event that adequate rate control cannot be achieved with either a beta blocker or a combination of a beta blocker and digoxin, then amiodarone (A) may be considered. Amiodarone can be used alone or in combination with other rate-slowing agents. Amiodarone is not recommended as a chronic rate-control medication, but in the acute setting can assist with rate control in a patient who cannot tolerate other therapies.

A patient is being discharged from the hospital after having a ST-elevation myocardial infarction. During his stay, he underwent percutaneous coronary intervention with placement of a drug eluting stent. The patient is being sent home on the following medications: aspirin 81 mg, metoprolol 50 mg, nitroglycerin 0.4 mg sublingual, and atorvastatin 40 mg. Which of the following should also be added to his regimen? Clopidogrel Fish oil Ranolazine Reteplase

Clopidogrel Clopidogrel (Plavix®) is a platelet P2Y12 receptor blocker. Similar agents include ticagrelor (Brillenta®) and prasugrel (Effient®). Dual antiplatelet therapy with aspirin and a platelet P2Y12 receptor blocker (such as clopidogrel) decreases the risk of coronary artery stent thrombosis and its consequences of myocardial infarction or death more than the use of aspirin alone. The duration of therapy of dual antiplatelet therapy differs for each type of stent. The current recommendation is treatment with a platelet P2Y12 receptor blocker for at least one year after placement of a drug eluting stent and one month after bare metal stent. Aspirin should be continued indefinitely for all stented patients. Fish oil (B) is sometimes recommended for primary and secondary prevention of coronary artery disease but is not essential for post percutaneous coronary intervention therapy. Ranolazine (C) is an anti-anginal medication that is added only after calcium channel blockers, beta-blockers, and nitrates have failed to control angina. Reteplase (D) is a fibrinolytic agent used to treat ST-elevation myocardial infarctions when a catheterization lab is not readily available. None of these agents play a role in preventing stent thrombosis.

Which of the following medications lowers cholesterol by binding bile acids and forming insoluble complexes that are excreted in the feces? Colesevelam Fenofibrate Gemfibrozil Lovastatin

Colesevelam Colesevelam, a bile acid sequestrant, decreases cholesterol absorption by forming insoluble complexes of bile acids that can then be eliminated through feces. Dyslipidemia is condition of elevated levels of plasma cholesterol or triglycerides or low levels of high-density lipoprotein (HDL), or a combination of these findings. Dyslipidemia is a major contributing risk factor for development of coronary artery disease (CAD). Reducing elevated cholesterol has been shown to reduce morbidity and mortality in patients with or without CAD. HMG-CoA reductase inhibitors, or statins, are the most commonly used class of cholesterol lowering drugs. However, some patients are unable to tolerate statins due to gastrointestinal complaints or myalgias. Statins are contraindicated in pregnancy, breastfeeding, active liver disease, myopathy, or unexplained elevated liver enzymes. When statins are intolerable or contraindicated, other classes of lipid-lowering drugs such as bile acid sequestrants, niacin, and fibric acid derivatives can be used. Bile acid sequestrants include cholestyramine, colestipol, and colesevelam. Bile acid sequestrants interrupt reabsorption of bile acids, resulting in lowered intrahepatic cholesterol. Maximum benefit is typically seen within 3 weeks. These agents are effective in patients with mild to moderate LDL elevation. Bile acid sequestrants can also be used in combination with statins or niacin in patients with severely elevated LDL levels. The main side effects of bile acid sequestrants are nausea, bloating, flatulence, abdominal pain, and constipation. Bile acid sequestrants can also cause a mild increase in triglyceride levels, and should be used with caution in patients with elevated baseline triglyceride levels. Bile acid sequestrants should be taken with meals because they block cholesterol absorption in the gastrointestinal tract. Because bile acid sequestrants can impair absorption of other drugs and fat-soluble vitamins, other medications should be taken one hour before or four hours after taking bile acid sequestrant. Colesevelam is pregnancy class B. Fenofibrate (B) and gemfibrozil (C) are fibric acid derivatives. The fibric acid derivatives mechanism of action is not completely known. It is thought that fibric acid derivatives stimulate lipoprotein lipase, which increases the catabolism of triglyceride-rich lipoproteins. Fibric acid derivatives main use is to lower serum triglycerides. They have minimal effect on LDL levels. Fibric acid derivatives are generally well tolerated. Lovastatin (D) is an HMG-CoA reductase inhibitor, or a statin. Statins inhibit the rate-limiting enzymatic production of cholesterol in the liver. By preventing the conversion of cholesterol, the liver increases the amount of LDL receptors that can take up LDL cholesterol from the blood.

A 26-year-old woman presents with dizziness and palpitations. She reports episodes of these symptoms beginning about 1 week ago, which initially only lasted a few minutes. However, for the past two days, she has had about 4 episodes a day which last about 20 minutes each. Her social history is significant for heavy caffeine intake. Her pulse is 166 bpm and her blood pressure is 140/70. Her rhythm strip is seen above. Which of the following best describes the most likely underlying cardiac pathology responsible for this patient's symptoms? Conduction reentry Dilated cardiomyopathy Myocardial infarction Valvular abnormality

Conduction reentry Paroxysmal supraventricular tachycardia (PSVT) is an acute onset tachydysrhythmia that can affect any age group. The dysrhythmia is caused by a reentrant conduction pathway, most commonly in the atrium or atrioventricular node. Common triggers include atrial or atrioventricular premature beats, hyperthyroidism, caffeine and drugs. The common symptoms of paroxysmal supraventricular tachycardia include palpitations, dizziness, dyspnea, syncope, angina, fatigue, nausea and sweating. The most common symptoms are palpitations and dizziness. The chest pain which accompanies this class of dysrhythmias is usually due to the tachycardia itself. Hemodynamically stable patients should first attempt vagal maneuvers. If unsuccessful, AV-nodal blocking agents can be administered such as adenosine or calcium channel blockers. If the patient is hemodynamically unstable, she should immediately undergo cardioversion. Dilated cardiomyopathy (B) and valvular abnormalities (D) are more affiliated with atrial fibrillation. Myocardial infarction (C) is more closely associated with ventricular tachycardia or fibrillation.

The emergency department staff began treatment for a woman who presented with chest pain. The pain is described as retrosternal, worse with minimal activity, better with rest, sharp in character and 9/10 in intensity. You are paged to admit her to the intensive care unit under the working diagnosis of unstable angina. Her vitals have remained stable after beginning antiplatelet, antihypertensive and antithrombotic medications. Two hours after admission, a repeat history and physical and review of available test results offers the following information: Serial electrocardiograms reveal increasing R wave amplitude; An echocardiogram calculates an ejection fraction of 50%; Angina is reported as 9/10 in intensity; Atrial natriuretic peptide levels are elevated. Which of the following historical facts would prompt you to immediately consult interventional cardiology for invasive coronary revascularization? 50% ejection fraction Continued chest pain Elevated atrial natriuretic peptide R wave progression

Continued chest pain Patients with unstable angina are mostly admitted to a critical care unit after initial presentation. There, an anti-ischemic regimen, if not already begun, is initiated. This typically includes oxygen, nitrates, analgesics and beta-blockers. Serial monitoring for new dysrhythmias, recurrent ischemia, dynamic electrocardiography, changing laboratory results and worsening angina is necessary to maximize patient outcomes. Further management includes risk stratification to determine if early invasive treatment is appropriate. High-risk indicators that favor early invasive treatment strategies include hemodynamic instability, elevated troponin I or T levels, a history of CABG, a history of percutaneous coronary intervention (PCI) within the past 6 months, recurrent angina despite anti-ischemic therapy, symptoms of congestive heart failure (S3, pulmonary edema, crackles, mitral regurgitation) or an ejection fraction < 40%. An ejection fraction < 40%, not 50% (A), favors early invasive treatment of unstable angina. Whereas elevated B-type natriuretic peptide is associated with poor outcomes in patients with unstable angina, atrial natriuretic peptide (C) is not. It is however responsible for water, sodium and potassium homeostasis. Its action is opposite of aldosterone. R wave progression (D) is not indicative of invasive management of unstable angina. New or presumably new ST depression is, however.

A 4-year-old boy presents with a 5-day history of fever. On exam he is noted to have non-tender anterior cervical lymphadenopathy, non-exudative conjunctival injection, and swollen hands and feet. Which of the following is the most common complication of this condition? Aplastic anemia Coronary artery aneurysm Glomerulonephritis Severe thrombocytopenia

Coronary artery aneurysm Kawasaki disease (KD), also known as mucocutaneous lymph node syndrome, is one of the most common vasculitides in children. It typically presents in children younger than 4 years, most commonly around age 2 and has a male predominance. Clinical manifestations are characterized by fever of at least five days duration. In addition, there must be at least four of the following: erythema of the lips and oral mucosa, conjunctivitis, cervical lymphadenopathy, and edema or erythema of the extremities. The disease is self-limited, however a complication of Kawasaki disease is coronary artery aneurysm. Therefore, when the diagnosis is suspected, prompt treatment is critical. A cardiac echocardiogram should be performed at the time of diagnosis, 2-3 weeks later, and 6-8 weeks after the onset of illness. KD is treated initially with IVIG and high dose aspirin followed by maintenance aspirin. Aplastic anemia (A) is a known complication of several infections such as human parvovirus B19 or HIV, however it is not associated with Kawasaki disease. Glomerulonephritis (C) is a sequela of streptococcal pharyngitis. Kawasaki disease is associated with thrombocytosis by the fifth day of the illness, not thrombocytopenia (D).

A 43-year-old man is currently undergoing palliative radiation therapy for pancreatic cancer. He presents to the emergency department with a 4-day history of gradually worsening left lower extremity pain. Pain is localized to the calf and made worse with ambulation. Exam shows mild edema of the left calf without erythema or warmth. Which of the following is the most likely diagnosis? Cellulitis Deep venous thrombosis Peripheral artery disease Superficial thrombophlebitis

Deep venous thrombosis Deep venous thrombosis (DVT) is a formation of a thrombus in a deep vein in the body. They can occur anywhere in the deep venous system, but is most common in the deep veins of the legs. Symptoms and physical exam are often nonspecific, but may include unilateral limb swelling and pain, erythema, warmth, distended collateral veins, and a tender palpable cord. Some predisposing risk factors include malignancy, immobilization, recent surgery, obesity, smoking, oral contraceptives, recreational drugs, and hypercoagulable states. The diagnosis should be considered in all patients presenting with a clinical presentation suspicious for DVT. The diagnosis is usually made through duplex ultrasound. Serum lab testing (D-dimer), venography, CT angiogram, and MRI are other diagnostic modalities. Treatment is with anticoagulation with agents such as low molecular weight heparin. Cellulitis (A) is an acute infection of the skin that is associated with erythema, swelling, pain and warmth that is localized to a specific area. Patients may also have systemic symptoms of infection including fever or malaise. Peripheral artery disease (C) causes pain with ambulation that is relieved with rest (intermittent claudication) and is associated with skin color changes, hair loss, and numbness or weakness. Superficial thrombophlebitis (D) is a thrombus in a superficial vessel that causes localized pain, erythema, and warmth. There is often a tender, nodular cord that can be palpated to help confirm this clinical diagnosis.

A bicuspid aortic valve is often associated with which of the following findings? Abdominal aortic aneurysm Dilation of the ascending aorta Left atrial dilation Patent foramen ovale

Dilation of the ascending aorta Dilation of the ascending aorta is frequently associated with a bicuspid aortic valve. Bicuspid aortic valve disease is one of the most common congenital heart lesions, occurring in approximately 1% of the population. Bicuspid aortic valve has an increased prevalence associated with congenital lesions such as coarctation of the aorta or Turner syndrome. More than 70% of patients with a bicuspid valve will require surgical intervention for a stenotic or regurgitant valve or aortic pathology over the course of a lifetime. Dilation of the ascending aorta in the setting of a bicuspid valve was previously considered a secondary event due to abnormal aortic valve function; however, it is now recognized to be caused by intrinsically abnormal connective tissue. As a result, serial evaluation of ascending aortic diameter should be performed by transthoracic echocardiography. An abdominal aortic aneurysm (A) is not associated with a bicuspid aortic valve. This condition is more common in men with a history of smoking and chronic hypertension. A patent foramen ovale (D) is a congenital cardiac lesion connecting the left and right atria that frequently persists into adulthood; however it is not associated with a bicuspid aortic valve. Left atrial dilation (C) occurs in association with chronic atrial fibrillation, not bicuspid aortic valve. Dilation can promote atrial fibrillation by perturbing propagation of orderly depolarization. Conversely, permanent atrial fibrillation can promote left atrial dilation.

Which of the following is a late complication of acute myocardial infarction? Cardiogenic shock Complete heart block Dressler syndrome Pulmonary edema

Dressler syndrome Dressler syndrome is an autoimmune mediated acute febrile illness associated with a pericarditis that occurs two weeks to several months after an acute myocardial infarction. It may also occur after surgery or traumatic myocardial injury. Symptoms include fever, cough, palpitations and chest pain, which may be similar to chest pain experienced during a myocardial infarction (MI). The postulation is that after a myocardial infarction, antigens to myocardial cells are released into the pericardial fluid which trigger the autoimmune process. Dressler syndrome is seen in approximately 5% of patients post-MI. Large pericardial effusions may occur leading to cardiac tamponade. Complete heart block (B) may occur within hours of an acute myocardial infarction (AMI). This is most commonly associated with an inferior wall MI because the right coronary artery which is most commonly affected supplies the inferior wall of the heart as well as the sino-atrial node. It is usually transient and may require a temporary transvenous pacemaker. Cardiogenic shock (A) occurs within hours of an AMI. This is characterized by hypotension and signs of heart failure. These patients may require inotropic support and or an intra-aortic balloon pump. Pulmonary congestion and pulmonary edema (D) can occur within hours if there is sufficient insult to the myocardium. In severe cases patients may require noninvasive positive pressure ventilation or invasive ventilation support.

Which of the following is the most common location of aortoenteric fistula formation? Duodenum Esophagus Jejunum Sigmoid colon

Duodenum An aortoenteric fistula is an abnormal communication between the aorta and the gastrointestinal tract. Primary aortoenteric fistulas are caused by compression of gastrointestinal structures by an aortic aneurysm. Secondary aortoenteric fistulas result from erosion of an aortic prosthetic graft into an adjacent gastrointestinal structure. As such, abdominal aortic aneurysm and a history of aortic surgery are the most common risk factors for aortoenteric fistula formation. Other causes include reflux esophagitis, peptic ulcer disease, non-aneurysmal aortitis, and penetrating aortic ulcers. The duodenum is the most common site of fistula formation. The classic triad of gastrointestinal bleeding, abdominal pain, and a palpable mass is rarely present and a known history of aortic aneurysm is often lacking. Gastrointestinal bleeding, including hematemesis, hematochezia, and melena, is often the presenting symptom. Although massive hemorrhage is common, many patients will have a small "herald bleed," a seemingly self-limited episode of gastrointestinal bleeding, prior to a larger bleed. Complications include hemorrhage and sepsis from seeding of the blood with gastrointestinal flora. While uncommon, aortoenteric fistula should be on the differential diagnosis for gastrointestinal bleeds, especially in patients with a history of aneurysm or aortic surgery, as the condition is life-threatening. Management of aortoenteric fistulas is surgical repair. The most common location of aortoenteric fistula formation is the duodenum, but aortoenteric fistulas can form anywhere along the gastrointestinal tract, including the esophagus (B), stomach, jejunum (C) and sigmoid colon (D).

What is the most common symptom of cardiac ischemia in patients older than 85 years?

Dyspnea.

Which of the following signs or symptoms is most likely to be associated with a patient with chronic venous insufficiency? Edema Hair loss Muscle atrophy Pallor

Edema Chronic venous insufficiency is the inability of the extremities to return blood to the heart which leads to a pooling in the lower extremities and edema. Clinically, it may present as discomfort or heaviness to the thigh or calf especially when standing or walking. Varicose veins may be present in superficial areas as blood is shunted from deep veins to superficial veins. Venous insufficiency may lead to thrombosis, the precursor to deep vein thrombosis or pulmonary embolism. The condition is diagnosed with venous duplex ultrasound. First line treatment includes: diet and exercise, compression stockings and elevation of the affected limb above the level of the heart. Pharmacologic therapy may include sulodexide which is a potent, long acting antithrombotic. Surgically the condition may be treated with ligation and stripping, sclerotherapy or radiofrequency ablation. Hair loss (B), muscle atrophy (C) and pallor (D) all occur in the setting of peripheral artery disease which differs from venous insufficiency in that oxygenated blood is not reaching the extremity. Ischemia can cause pain, pulselessness, coolness to the touch, paresthesia, cyanosis and necrosis of the foot and digits.

A 72-year-old man has been more fatigued lately. As a result, his primary practitioner orders an electrocardiogram. A P wave is present for every QRS complex and the PR interval is 225 milliseconds in length. Which of the following is the diagnosis? First degree atrioventricular block High-grade atrioventricular block Mobitz type I atrioventricular block Third degree atrioventricular block

First degree atrioventricular block A first degree atrioventricular (AV) block is characterized by a long PR interval (>200 milliseconds). The normal PR interval is 120 to 200 milliseconds in length. The PR interval represents the time between atrial depolarization (P wave) and the ventricular depolarization (QRS complex). A prolonged PR interval is a sign of a delayed conduction at the AV node before ventricular depolarization. Causes of first degree AV block include a benign increase in vagal tone, drugs that slow AV conduction (e.g., beta-blockers, digoxin), previous myocardial infarction, dilated cardiomyopathy, and some types of muscular dystrophy. Typically, patients exhibit no symptoms and there are no signs on physical exam. A high grade atrioventricular block (A) is defined as second degree heart block (either Mobitz type I or Mobitz type II) with a P:QRS conduction ratio of 3:1 or higher. It results in a very slow ventricular rate. Mobitz type I heart block (C) is defined as a progressive lengthening of the PR interval that eventually results in a P wave without an associated QRS complex. Third degree atrioventricular block (D) is complete dissociation between the atria and the ventricles. This will result in P waves that are not associated with QRS complexes.

Which of the following is a class IC anti-arrhythmic? Amiodarone Flecainide Lidocaine Procainamide

Flecainide Flecainide is a class IC anti-arrhythmic drug. All class I anti-arrhythmics exert their action at the fast sodium channels. The subsets of class I relate to how the drugs affect depolarization, repolarization and conduction. The class IC drugs slow depolarization and conduction. Flecainide is most commonly used in the treatment of supraventricular tachycardias as well as ventricular tachycardia not related to acute ischemia. Procainamide (D) is a class IA drug that is the most common drug in this class used for treatment of supraventricular tachycardias and ventricular tachycardias in the emergency setting. Class IA agents slow conduction through the atria, AV nodes and His-Purkinje systems. Additionally, conduction through accessory pathways is slowed. Infusion of procainamide may lead to hypotension and QRS widening. Lidocaine (C) is the only Class IB agent used in emergent treatment of dysrhythmias particularly those resulting from increased automaticity like ventricular tachycardia. These agents work by slowing conduction and depolarization and shortening repolarization. Amiodarone (A) is a class III drug, although it also has characteristics of Class IA, II and IV agents. The class III drugs prolong the refractory period by blocking potassium channels.

What cardiac complication is associated with hyperthyroidism? Aortic Dissection High-output cardiac failure Pericarditis Ventricular dysrhythmias

High-output cardiac failure High-output cardiac failure is associated with hyperthyroidism. Hyperthyroidism is caused by an increased circulating level of thyroid hormone. Elevated thyroid hormone levels produce a hypermetabolic state and increased beta-adrenergic activity. Common cardiac complaints include palpitations, dyspnea on exertion and decreased exercise tolerance. New-onset atrial fibrillation is common especially in the elderly. Through beta-adrenergic stimulation, elevated thyroid hormone levels can produce high-output cardiac failure. Dilated cardiomyopathy can result from prolonged tachycardia (or tachydysrhythmias) along with the high output state. Aortic dissection (A) and pericarditis (C) are not associated with hyperthyroidism. Atrial dysrhythmias, not ventricular (D) are associated with hyperthyroidism.

What is the most common side effect following administration of intravenous amiodarone? Bradycardia Hypotension Nausea Widening of the QRS interval

Hypotension Amiodarone is used in the treatment of both ventricular and supraventricular dysrhythmias. It is classified as a class III antiarrhythmic drug due to is inhibition of outward potassium channels which prolongs the duration of the action potential. It also has properties of class I antiarrhythmics (sodium channel blockage), class II antiarrhythmics (beta adrenergic receptor blockage) and class IV antiarrhythmics (calcium channel blockage). It has superior efficacy compared to many other antiarrhythmic drugs and a low rate of ventricular proarrhythmia. It is not, however, without side effects. The most common side effect of intravenous administration is hypotension, which can occur in up to a quarter of patients and may be attributed to the solvents used in the preparation. Bradycardia (A) is another potential side effect of intravenous amiodarone use and occurs in less than 5% of patients. Bradycardia is more likely when amiodarone is given in conjunction with other antiarrhythmic drugs such as calcium channel blockers and beta blockers. Nausea (C) is an uncommon side effect of intravenous administration of the drug but is more likely with oral use. Intravenous use of amiodarone results in a smaller increase in the duration of the action potential compared with oral use resulting in little to no widening of the QRS interval (D).

A 65-year-old man presents to the ED with a known history of heart failure. He complains of progressive shortness of breath over the preceding month to the point that he now has to rest even when he walks from his bed to the bathroom. These symptoms resolve at rest. What is this patient's New York Heart Association classification? I II III IV

III The New York Heart Association functional classification scheme is used to assess the severity of functional limitations in patients with chronic heart failure and correlates fairly well with prognosis. Patients in Class III have moderate limitations and have symptoms of heart failure with minimal activity such as walking across a room. Class I (A) is associated with no limitation and no symptoms during usual daily activities. Class II (B) has slight limitations with mild symptoms (dyspnea, fatigue, or chest pain) with ordinary daily activities. Class IV (D) has severe limitation with symptoms of heart failure occurring at rest.

Which of the following is the first line treatment for acute pericarditis? Citalopram Ibuprofen Ketorolac Prednisone

Ibuprofen If there is a specific etiology of the pericarditis, then therapy should be directed at that etiology. Otherwise, nonsteroidal anti-inflammatory drugs (NSAIDs) are the mainstay of therapy for acute pericarditis. Ibuprofen has the best side effect profile, but other NSAIDs should be equally effective.The majority of patients have prompt resolution of symptoms without recurrent pericarditis when treated with NSAIDS alone. However, when used as an adjunct to NSAID therapy, colchicine reduces symptoms, decreases the rate of recurrent pericarditis, and is generally well tolerated. Citalopram (A) is a serotonin-reuptake inhibitor (SSRI) and has no role in the treatment of acute pericarditis. Ketorolac (C) is an NSAID with an excellent analgesic profile, but it has poor anti-inflammatory effects. Therefore, it is not recommended for the treatment of acute pericarditis. Prednisone (D) is a corticosteroid with anti-inflammatory properties that has use limited to the treatment of refractory pericarditis (patients who are intolerant or unresponsive to NSAIDS and/or colchicine). There is a risk of relapse of symptoms when the steroids are tapered or stopped.

What is the most common cause of pericarditis?

Idiopathic.

A 44-year-old woman presents to the Emergency Department with palpitations following the death of her mother 2 days prior. Her ECG is consistent with normal sinus rhythm with a QTc interval of 520 msec. Her heart rate is 86 and blood pressure is 117/82. Her medications include lisinopril and metformin. She took lorazepam yesterday for anxiety related to the loss of her mother. What is the most appropriate therapy? Administer adenosine Discontinue lorazepam Initiate metoprolol Perform vasovagal maneuvers

Initiate metoprolol Long QT syndrome is defined by a QT interval of more than 460 msec for women and more than 440 msec for men on ECG. It is more commonly caused by medications than by familial prolongation. Physical and emotional stress are common triggers. Manifestations include palpitations, syncope and cardiac arrest. Beta-blockade with medications such as metoprolol may be appropriate in patients who are hemodynamically stable. Cardiology consult or referral is recommended because implantation of a cardiac defibrillator can be beneficial for those at risk of sudden cardiac death. Adenosine (A) is recommended in patients with palpitations due to supraventricular tachycardia. Lorazepam (B), however, is not associated with QT prolongation. Vasovagal maneuvers (D) are recommended in patients with palpitations due to supraventricular tachycardia.

Which of the following is a mainstay for the treatment of Kawasaki disease during the acute febrile phase? Aspirin - low dose Intravenous immunoglobulin (IVIG) Steroids Warfarin

Intravenous immunoglobulin (IVIG) Intravenous immunoglobulin (IVIG), as well as high dose aspirin, form the two components of the treatment of Kawasaki disease during the acute febrile phase of the disease. IVIG involves an infusion of 2 g/kg over 10 to 12 hours. High dose aspirin (80-100mg/kg/day divided into 6 hour dosing regimens) is continued until the child is afebrile for 48 to 72 hours or longer. Low dose aspirin (A) (3 to 5 mg/kg/day) is then initiated and continued until the patient shows no coronary changes by 6 to 8 weeks. The benefit of steroids (C) is controversial and current guidelines recommend restricting steroid treatment to children in whom >2 infusions of IVIG have been ineffective in alleviating fever and acute inflammation. When a patient with Kawasaki disease develops coronary artery disease, treatment may involve antiplatelet therapies (aspirin, with or without dipyridamole or clopidogrel), anticoagulant therapy with warfarin (D) or low-molecular-weight heparin, or a combination of anticoagulant and antiplatelet therapy (usually warfarin plus aspirin). No prospective data exist to guide clinicians in regards to the optimal regimen.

A man presents to the emergency department with a blood pressure of 200/136 mm Hg and laboratory evidence of acute renal failure. Which of the following medications and initial blood pressure response rates are the most appropriate at this time? Intravenous labetalol, to goal within 1 to 2 hours Intravenous nicardipine, to goal within 6 to 12 hours Oral clonidine, to a goal within 3 to 6 days Oral magnesium sulfate, to a goal within 1 to 2 days

Intravenous labetalol, to goal within 1 to 2 hours A hypertensive emergency is a severe elevation in blood pressure with evidence of end-organ damage. This requires immediate lowering of blood pressure. In hypertensive emergencies, treatment begins in the emergency department and continues in the intensive care unit. It begins with obtaining intravenous access for medication delivery, obtaining intra-arterial access for monitoring and serial measurements of urinary output, blood pressures, mental status, and renal indices. Medication options include labetalol, esmolol, nicardipine, hydralazine, fenoldopam, nitroglycerin, nitroprusside, and phentolamine. IV labetalol (a beta-adrenergic and alpha-adrenergic blocker) has a rapid onset of action (within minutes), making it very useful for the treatment of hypertensive emergencies. In most hypertensive emergencies, the mean arterial pressure (MAP) should be reduced by about 10-20% in the first hour, followed by a gradual reduction during the next 23 hours. The treatment goal is to achieve a final pressure reduction by 25% compared with baseline. In hypertensive emergencies, the goal of treatment is to decrease the mean arterial pressure by 25% over minutes to 2 hours, with an ultimate goal of obtaining diastolic stability < 110 mm HG within 2 to 6 hours (B). Acute exacerbation of chronic hypertension is treated with oral medications to a goal of normal blood pressure over 1-2 days (C). Hypertensive crisis during pregnancy is likely part of eclampsia, a seizure-hypertension syndrome that occurs in 12% of pregnancies. Initial treatment includes volume expansion, intravenous antihypertensives (namely labetalol or nicardipine), seizure prophylaxis with magnesium sulfate (D) and immediate delivery.

What is a Holter monitor?

It is an ambulatory ECG worn for 24-48 hours and it is the most frequent non-invasive test used to evaluate cardiac rhythm abnormalities.

A 28-year-old woman with a history of intravenous drug use presents to the emergency department with complaints of chest pain, shortness of breath, cough and fever. Which of the following physical exam findings is most suggestive of infective endocarditis? Hutchinson's sign Janeway lesions Koplik's spots Levine's sign

Janeway lesions Infective endocarditis (IE) is an infection of the endocardial surfaces of the heart caused by a number of different etiologies and pathogens. Native valve endocarditis is caused by rheumatic valvular disease, mitral valve prolapse, congenital heart disease or degenerative heart disease. Prosthetic valve endocarditis presents shortly after valve replacement surgery. Intravenous drug abuse endocarditis most commonly affects the tricuspid valve with Staphylococcus aureus being the most common pathogen responsible. Patients often present with nonspecific complaints of fever, chills, night sweats, myalgias, joint pain, anorexia, and weight loss. Classic signs of IE include petechiae, subungual hemorrhages, tender nodules on the fingertips, and nontender macules on the palms and soles, commonly referred to as Janeway lesions. Hutchinson's sign (A) refers to vesicles that develop on the nose related to herpes zoster infection. Koplik's spots (C) are small white spots seen in the mouth of a patient with measles. Levine's sign (D) describes a patient clenching a fist over their chest in response to ischemic chest pain.

Chronic Heart Failure Treatment

Lifestyle modifications • Diuretics: used for acute pulmonary edema, no mortality benefit • ACEIs: ↓ mortality in all classes • BBs: ↓ mortality in classes II, III, IV • Hydralazine with nitrates: ↓ mortality in African-Americans • Digoxin: used in refractory systolic dysfunction, no mortality benefit • Spironolactone: ↓ mortality in class III/IV • Isolated diastolic dysfunction: HR and BP control • Advanced treatments: mechanical assist device, ICD, heart transplantation

Which of the following would you most expect to find in a patient who presents to the emergency department in cardiogenic shock? Dehydration High pulse pressure Low cardiac index Warm, hyperemic extremities

Low cardiac index. Cardiogenic shock is the leading cause of death in acute myocardial infarction. It is characterized by decreased cardiac output and tissue hypoxia in the presence of sufficient intravascular volume. Patients present with hypotension, tachycardia, altered mentation, cool cyanotic extremities, faint peripheral pulses, and oliguria. A low pulse pressure is also typically encountered. Coronary angiography is indicated if myocardial ischemia or infarct is present. Invasive hemodynamic monitoring may be used, the results of which show a high pulmonary capillary wedge pressure (> 15 mm Hg) and a low cardiac index (stroke volume x heart rate / body surface area; < 2.2 L/min/m2). The cardiac index is a hemodynamic parameter that relates the cardiac output from the left ventricle in one minute to body surface area. Thus, relating cardiac performance to the size of the individual. Treatment mainstay includes prompt inotropic medications such as dopamine, dobutamine and phosphodiesterase inhibitors, and norepinephrine. Other measures include the placement of central and peripheral arterial lines, possible fluid resuscitation, ICU care, electrolyte and acid-base correction, intra-aortic balloon pump, percutaneous coronary intervention or coronary artery bypass grafting. Dehydration (A), as evidenced by orthostatic hypotension and dry mucous membranes, are not expected in cardiogenic shock, as the definition of this type of shock excludes the possibility of hypovolemia. A low, not high, pulse pressure (B) is expected in cardiogenic shock. Pulse pressure is the systolic BP minus the diastolic BP, normally considered around 40 mm Hg (120-80). Low pulse pressure is considered when the pulse pressure is < 25% of the systolic BP. Cool, cyanotic extremities (D) are expected in cardiogenic shock.

A woman in her second trimester of pregnancy develops a left lower extremity deep vein thrombosis. Which of the following is the most appropriate choice of anticoagulation for this patient? Apixaban Low molecular weight heparin Unfractionated heparin Warfarin

Low molecular weight heparin Low molecular weight heparin is recommended at this stage of this woman's pregnancy for prevention of thromboembolism from her deep vein thrombosis. Pregnancy is a hypercoaguable state, and anticoagulation during pregnancy is indicated for women with atrial fibrillation, mechanical valve prosthesis, antiphospholipid antibody syndrome and venous thromboembolism. Unfractionated heparin, and low molecular weight heparin are approved for use during pregnancy. In contrast to anticoagulation of non-pregnant women, the choice of anticoagulant during pregnancy needs to take into account fetal safety and maternal peripartum issues such as unpredictable onset of labor. Heparins are used for most pregnant women because they do not cross the placenta and do not result in fetal anticoagulation. Low molecular weight heparin is recommended over unfractionated for all but the final weeks of the pregnancy, because they are effective and easier to administer than unfractionated heparin. Low molecular weight heparin produces a more predictable anticoagulant response than unfractionated heparin and do not require routine monitoring. Unfractionated heparin is a reasonable alternative to a low molecular weight heparin when cost or need for rapid reversal is important such as for delivery. Apixaban (A) is an oral factor Xa inhibitor. This and other newer anticoagulation agents are avoided during pregnancy because of absence of information on efficacy and fetal safety. Warfarin (D) is avoided during pregnancy because it crosses the placenta, is a teratogen, and causes fetal anticoagulation throughout the pregnancy. Exposure during early pregnancy can result in embryopathy, while exposure later in pregnancy can cause fetal bleeding, including intracranial hemorrhage. Unfractionated heparin (C) is generally only recommended in the final weeks of the pregnancy because it can be rapidly reversed for delivery. Since it is administered intravenously (unlike low molecular weight heparin which is administered subcutaneously), it is not feasible to maintain an individual on this therapy throughout a pregnancy.

What is phlegmasia cerulean dolens?

Massive thrombosis causing venous insufficiency (painful blue leg).

Which of the following statements is true regarding prosthetic heart valves? Anticoagulation is optional with mechanical valves Mechanical valves are associated with less hemolysis and are less thrombogenic than bioprosthetic are Mechanical valves are more prone to paravalvular leaks than bioprosthetic valves are Mechanical valves make opening and closing sounds similar to, but louder than, those of native valves

Mechanical valves are more prone to paravalvular leaks than bioprosthetic valves are Prosthetic heart valves are classified as either mechanical (constructed of entirely synthetic material) or biologic (human, porcine, bovine). All prosthetic valves are associated with complications ranging from structural failure and thrombosis to systemic embolization, hemolysis, and endocarditis. A paravalvular leak occurs when a portion of the prosthetic valve becomes unseated from the valve annulus. It is more common with mechanical valves. When it occurs immediately after surgery, it is usually due to suture disruption, whereas delayed leaks are generally due to endocarditis. Clinically, patients typically present with sudden onset of pulmonary edema or severe hemolytic anemia. It is often associated with a regurgitant murmur. Bioprosthetic valves have a lifespan of 8-10 years; this is less than the > 20-year lifespan of mechanical valves. Mechanical valves are associated with greater hemolysis (B) and are more thrombogenic than bioprosthetic valves are and therefore require lifelong anticoagulation (A). Prosthetic valve thrombosis has an incidence of approximately 2% per year, with both bioprosthetic and appropriately anticoagulated mechanical valves. Mechanical valves typically make a loud metallic closure sound and a softer opening click (D). Mechanical aortic valves are also associated with a systolic ejection murmur. Bioprosthetic valves make opening and closing sounds that are similar to, but louder than, native valves.

A 33-year-old woman is seen in clinic for pregnancy induced hypertension. Which of the following antihypertensives is considered safe during pregnancy? Lisinopril Losartan Methyldopa Nitroprusside

Methyldopa Methyldopa is a drug of first choice for control of mild to moderate hypertension in pregnancy and is the most widely prescribed antihypertensive for this indication. Gestational hypertension refers to elevated blood pressure first detected after 20 weeks of gestation in the absence of proteinuria or other diagnostic features of preeclampsia. When hypertension is diagnosed in a pregnant woman, the major issues are establishing a diagnosis, deciding the blood pressure at which treatment should be initiated and avoiding drugs that may adversely affect the fetus. Methyldopa has been widely used in pregnant women and its long-term safety for the fetus has been demonstrated. During long term use in pregnancy, methyldopa does not alter maternal cardiac output or blood flow to the uterus or kidneys and for all these reasons is generally considered the agent of choice for blood pressure control in pregnancy. Labetalol, hydralazine and long-acting nifedipine are also acceptable oral antihypertensive options. Angiotensin converting enzyme inhibitors such as lisinopril (A), angiotensin II receptor blockers such as losartan (B), and nitroprusside (D) are contraindicated in all stages of pregnancy because of the risk of teratogenicity and toxic side effects.

What is the classic auscultatory feature of mitral valve prolapse? Crescendo-decrescendo systolic murmur Loud S1 and an opening snap in diastole Midsytolic click Soft diastolic murmur

Midsytolic click The classic auscultatory features of mitral valve prolapse are a midsystolic "click" followed by a midsystolic to late systolic murmur over the mitral area. This click results from snapping of the chordae tendineae during the prolapse of the valve. Mitral valve prolapse (MVP) is defined pathophysiologically as an abnormal movement of one or both of the mitral valve leaflets across the plane of the valve during systole. The typical auscultatory findings should suggest MVP and can be confirmed by echocardiography. Symptoms attributed to MVP, however, are often not explained by the degree of prolapse or mitral regurgitation. Although generally a benign condition, it is infrequently associated with more serious cardiac pathology such as mitral regurgitation, endocarditis, and arrhythmias. Echocardiographic studies report a true prevalence of less than 1% in both men and women versus the previously reported 5% with a female predominance. Crescendo-decrescendo systolic murmur (A) is the classic auscultatory finding in aortic stenosis. In mitral stenosis auscultatory findings include a loud S1 and an opening snap (B) in early diastole accompanied by a low-pitched, rumbling diastolic apical murmur. A short, soft, diastolic murmur (D) may be difficult to detect an aortic insufficiency therefore an echocardiography may be required and is diagnostic.

A 68-year-old man was admitted to the hospital for an acute exacerbation of his chronic systolic congestive heart failure. What education should be given to this patient upon discharge to help prevent readmission? Avoid physical activity Elevate lower extremities Monitor daily weights Restrict fluid intake

Monitor daily weights Instructing patients to monitor daily weights can help prevent heart failure readmission. Heart failure is one of the most common causes of hospitalization, hospital readmission and death. Due to the complexity and long-term nature of heart failure regimens, the need for careful diet and weight management, and the importance of intervention in the early phases of decompensation, patient self-management is crucial in avoiding hospitalizations. Patients should be specifically instructed to take all medications as directed, monitor daily weights, monitor for signs and symptoms, adhere to a low-sodium diet, limit alcohol consumption and stop smoking. Elevation of the lower extremities (B) can help reduce edema but it will not help prevent readmission to the hospital with a congestive heart failure exacerbation. Heart failure patients should not be advised to avoid physical activity (A). In stable patients, increasing physical activity or regular exercise can help diminish symptoms. Lastly, restricting fluid intake (D) has not been shown to prevent heart failure readmissions.

A 33-year-old woman with no medical problems presents with chest pain and shortness of breath. The symptoms worsened over the previous two days and increase with exertion. The patient denies cough, but last week reports fevers, chills, coughing and myalgias. Her ECG demonstrates sinus tachycardia without ST segment changes. Vital signs are T 100.7°F, HR 120, BP 108/60, RR 16, 100% saturation on room air. Which of the following is the most likely diagnosis? Acute coronary syndrome Mediastinitis Myocarditis Pulmonary embolism

Myocarditis Myocarditis can be due to an infectious or non-infectious process, with most infectious etiologies caused by a viral infection, most commonly parvovirus B19, adenovirus, coxsackie B virus, and Echovirus. Trypanasoma cruzi (Chagas disease) is the most common etiology worldwide. Myocardial necrosis occurs likely as a result of direct invasion of the offending organism as well cytotoxic effects of the host's immune system. Individuals develop flulike symptoms and in adults chest pain and shortness of breath. One of the hallmark signs of myocarditis is tachycardia out of proportion to fever. Depending on the time of presentation, patients may have symptoms of heart failure as the left ventricular ejection fraction is impaired as a result of the myocarditis. Troponin is often elevated as the disease progresses. The gold standard for diagnosis is endomyocardial biopsy. Management is supportive. ACE inhibitors help reduce myocardial inflammation. Acute coronary syndrome (A) is a possibility in this patient but is less likely given her age and absence of medical history. It is also atypical for a person with acute coronary syndrome to have a fever and viral symptoms. Mediastinitis (B) may present with fever, chest pain and shortness of breath. However, there is often a history of either severe retching or a procedure that caused esophageal perforation. Patients with air in the mediastinum from a perforation may have Hamman's sign, a crunching sound on auscultation. Pulmonary embolism (D) does cause chest pain and shortness of breath. Classically the chest pain is pleuritic and flulike symptoms are not associated with the diagnosis. Patients with a pulmonary embolism may have a low-grade fever.

A 32-year-old woman is diagnosed with Prinzmetal's angina. She presents to clinic 2 weeks after being discharged from the hospital. She complains of 3 episodes of angina since discharge. She is concerned about future attacks. Long-term prophylaxis to reduce these attacks is best accomplished with which of the following medications? Atorvastatin Nifedipine Propranolol Warfarin

Nifedipine Prinzmetal's angina is a condition of cyclical chest pain secondary to coronary vasospasm. Any acute angina must be evaluated and empirically treated with an anti-ischemic regimen of antihypertensive, antiplatelet and antithrombotic medications. Once the diagnosis of coronary vasospasm is made, long term management of the anginal attacks include calcium channel blockers, namely nifedipine, and long-acting nitrates like isosorbide dinitrate. Statin medications (A) do play a role in the overall management of patients with Prinzmetal's angina, as there is some correlation with atherosclerosis. However, statins do not work at decreasing vasospasm, and as such, they would not be a viable choice in decreasing the chest pain attacks. Nonselective beta-blockers like propranolol (C) are contraindicated in the treatment of Prinzmetal's angina. Anticoagulants like warfarin (D) play no role in decreasing the vasospasm and subsequent angina associated with Prinzmetal's.

Which of the following is the most predictive risk factor for cardiac ischemia? Diabetes mellitus Family history of coronary artery disease Past medical history of coronary artery disease Tobacco use

Past medical history of coronary artery disease The most predictive risk factor for cardiac ischemia is a past medical history of coronary artery disease. The classical coronary risk factors have been repeatedly shown to increase the risk of ischemic heart disease in patients followed over many years. But in the acute setting, a history of coronary artery disease is most predictive. Diabetes mellitus (A), a family history of coronary artery disease (B), tobacco use (D), hypertension, and hyperlipidemia are traditional risk factors for coronary artery disease based on population studies and are predictive for the lifetime development of coronary artery disease, but only weakly predictive of the likelihood of acute ischemia. They can, however, help to predict poor outcomes in patients already diagnosed with acute coronary syndrome.

What physical exam finding is a hallmark of pericarditis?

Pericardial friction rub.

An eight-year-old girl was brought to the clinic due to chest pain. One week ago, she had rhinorrhea, abdominal pain, and fever. Later, she complained of stabbing chest pain that is worse with inspiration and relieved by sitting upright. On physical exam, heart rate is 120, respiratory rate is 20, blood pressure is100/80, with clear breath sounds and distant heart sounds. Which of the following is the most likely diagnosis? Costochondritis Gastroesophageal reflux Pericarditis Pulmonary embolism

Pericarditis The girl has signs and symptoms that are suspicious for pericarditis. There are several viral agents that can cause pericarditis like enteroviruses, influenza, adenovirus, respiratory syncytial virus, and parvovirus. The most common symptom of pericarditis is chest pain that is characterized as sharp or stabbing, positional, radiating, worse with inspiration and relieved by sitting upright or prone. Other nonspecific symptoms include cough, fever, dyspnea, abdominal pain, and vomiting. Clues to the diagnosis are physical exam findings of muffled or distant heart sounds, tachycardia, narrow pulse pressure, jugular venous distension, and a pericardial friction rub provide clues to the diagnosis of acute pericarditis. Pericarditis can be complicated by cardiac tamponade that is recognized by the excessive fall of systolic blood pressure (>10 mm Hg) with inspiration. Abnormal findings on ECG include low voltage QRS amplitude, tachycardia, and abnormalities of the ST segments, PR segments, and T waves. Chest X-ray findings may be normal if effusion is not present. On the other hand, there may be cardiac enlargement in the presence of an effusion. The most sensitive test to identify the size and location of a pericardial effusion is by echocardiography. The chest pain of costochondritis (A) is usually reproducible by palpation of the tender areas with upper costal cartilages most frequently involved. Gastroesophageal reflux (B) may present with pleuritic pain but more common symptoms are heartburn, regurgitation, dysphagia, and cough. Pulmonary embolism (D) usually presents with dyspnea followed by pleuritic pain and cough.

An 81-year-old man presents to the Emergency Department in respiratory distress. He is sitting upright and appears anxious, dyspneic, and diaphoretic. Vital signs show blood pressure of 190/110 mm Hg, heart rate of 130 beats/minute, respiratory rate of 35 breaths/minute, and oxygen saturation of 85% on room air. Which of the following physical examination findings most strongly suggest heart failure as the cause of his respiratory distress? Jugular venous distension Presence of a fourth heart sound Presence of a third heart sound Wheezing

Presence of a third heart sound Respiratory distress can result from numerous pathologic states, including obstructive airway disease (e.g. asthma and chronic obstructive pulmonary disease), decompensated heart failure, myocardial infarction, pneumonia, upper airway obstruction, tension pneumothorax, pulmonary embolism, fat embolism, and neuromuscular disease, among others. Emergency physicians must quickly determine the cause of respiratory distress in order to initiate appropriate treatment. Heart failure is a common cause of respiratory distress. A weakened or diseased left ventricle or one facing high systemic pressures cannot adequately pump blood and as a result, blood pools in the lungs, leading to pulmonary edema and clinical symptoms of congestive heart failure. Symptoms include dyspnea on exertion, dyspnea at rest, orthopnea, paroxysmal nocturnal dyspnea, and peripheral edema. However, these symptoms are seen in many conditions and cannot be used to distinguish congestive heart failure from other causes of dyspnea. Physical exam findings that suggest congestive heart failure include presence of a third heart sound or S3 gallop (likelihood ratio (LR) 11.0), hepatojugular reflux (LR 6.4), and jugular venous distention (LR 5.1). The combination of an S3 gallop and a chest radiograph showing pulmonary venous congestion or interstitial edema is highly suggestive of congestive heart failure. Jugular venous distention (A) is seen in congestive heart failure, but has a lower likelihood ratio than an S3 gallop. Presence of a fourth heart sound or an S4 (C) and wheezing (D) are not predictive of congestive heart failure.

A 40-year-old woman complains of recurrent chest pain that occurs shortly after she wakes up in the morning. She has a history of migraine headaches and Raynaud's phenomenon. She admits to tobacco abuse and smokes 1 pack-per-day. Her chest pain is not reproducible with palpation. An ECG during an episode reveals ST-elevation in multiple leads and cardiac biomarkers are normal. Which of the following is the most likely diagnosis? Costochondritis Panic disorder Prinzmetal angina Unstable angina

Prinzmetal angina Prinzmetal angina, or variant angina, is the most likely diagnosis in this patient. Prinzmetal angina is characterized by spontaneous episodes of angina in association with ST-segment elevation on ECG. The cause is a transient, abrupt, and marked reduction in the luminal diameter of an epicardial coronary artery due to spasm which leads to transient myocardial ischemia. Unlike most other causes of chest pain and ST-segment elevation, the ST-segment returns to baseline rapidly upon resolution of symptoms. This is a diagnosis of exclusion. Patients with variant angina are often younger and exhibit fewer classic cardiovascular risk factors, with the exception of cigarette smoking. Variant angina may be associated with other vasospastic disorders such as Raynaud's phenomenon and migraine headache or its treatment, such as sumitriptan. A history of drug abuse, such as cocaine, may also be present. Costochondritis (A) is musculoskeletal chest wall pain that causes more diffuse pain, in which multiple areas of tenderness are found and can usually be reproduced with palpation. Panic disorder (B) is a particularly common cause of chest pain; however it is not associated with ECG changes and is less consistent with this clinical picture. Unstable angina (D) is a presentation of acute coronary syndrome characterized by new, worsening or persistent chest pain, but in the absence of positive cardiac biomarkers. This can be difficult to distinguish from Prinzmetal angina; however, it would be less likely in a younger patient and it is not associated with other vasospastic disorders such as migraine headaches or Raynaud's phenomenon.

What side effect is responsible for the greatest number of deaths associated with amiodarone use?

Pulmonary toxicity, the most common of which is chronic interstitial pneumonitis.

A 19-year-old college student presents dead-on-arrival to the ED. A post mortem autopsy reveals significant calcification of the aortic valve cusps. Upon review of his medical history, you would most likely find documentation of a murmur heard in which of the following locations? Apex Left, fourth intercostal space Right, fourth intercostal space Right, second intercostal space

Right, second intercostal space Proper cardiac auscultation begins with an understanding of which chest wall location is associated with which valve-sound. The aortic valve is best appreciated in the right, second intercostal space just lateral to the sternum. The pulmonic valve is best heard in the left, second intercostal space just lateral to the sternum. The tricuspid valve can be appreciated in the left, fourth intercostal space over the left sternal border. The mitral valve is best appreciated in the left, fifth intercostal space about the midclavicular line, also known as the apex. The murmur of aortic stenosis occurs during systole between S1 and S2. As such, it is also called an ejection murmur. It is best heard in the right, second intercostal space. Other characteristics include medium pitch, crescendo-decrescendo tonality and possible associated thrill. Aortic stenosis is due to calcification of the valve cusps. It is a common cause of sudden death, especially in children and adolescents. It can also be associated with rheumatic heart disease, atherosclerosis and congenital bicuspid valve malformation. The mitral valve is best heard at the apex (A). An abnormality of the tricuspid valve would be best heard in the left, fourth intercostal space (B). The right, fourth intercostal space (C) does not have a specific valve-sound associated with it.

A 52-year-old man presents to the Emergency Department via ambulance with substernal chest pain. He received aspirin and nitroglycerin from emergency personnel in route. Vital signs include BP 95/55 mm Hg, HR 60 beats/minute, RR 20 breaths/minute, and T 97.6°F. His electrocardiogram is shown above. Which of the following is the most appropriate next diagnostic study? Chest radiograph Computed tomographic angiogram of the chest Posterior electrocardiogram Right-sided electrocardiogram

Right-sided electrocardiogram This patient has an ECG consistent with an inferior ST segment elevation myocardial infarction. Inferior wall myocardial infarction may be accompanied by bradycardia due to involvement of the AV node or hypotension due to total right ventricular involvement. The inferior wall and right ventricle are supplied by the right coronary artery. ST segment elevation in leads II, III, and aVF is suggestive of an inferior wall infarct. This patient is suffering from hypotension and bradycardia as well, which suggest a potential right ventricular infarct. ECG findings suggestive of this include ST segment elevation in leads II, III, and aVF with the elevation in lead III greater than that of lead II or associated elevation in lead V1. Any ST segment elevation in the inferior leads should prompt a right-sided electrocardiogram. ST segment elevation in leads V4R and V5R is diagnostic of a right ventricular infarct. Cardiac enzymes should also be drawn and troponin I would characteristically be elevated in ST segment elevation myocardial infarction 2-6 hours after symptom onset. Lack of positive troponin should not delay treatment. ST segment elevation in two contiguous anatomic leads should prompt immediate initiation of therapy. Aspirin 324 mg PO chewable should be administered immediately to prevent further platelet aggregation. Clopidogrel and heparin are also indicated. Nitroglycerin may be given to improve myocardial oxygen supply by dilating the coronary arteries; however, it is contraindicated in right ventricular infarct due to risk for profound hypotension. Definitive treatment for ST segment elevation myocardial infarction is cardiac catheterization and revascularization. If the patient is going to be greater than 90 minutes from medical contact to balloon time from the nearest cardiac catheterization center, thrombolytics are recommended. A chest radiograph (A) would be appropriate if you were concerned for occult pneumothorax or as an initial study to evaluate for widened mediastinum in aortic dissection. CT angiogram of the chest (B) would be appropriate as a definitive study if you were concerned for aortic dissection or pulmonary embolism. Posterior electrocardiogram (C) would be appropriate to evaluate for posterior ST segment elevation if you saw deep depression in leads V1 and V2 with prominent R waves.

A 61-year-old woman presents to the Emergency Department after several episodes of syncope. Her rhythm strip is shown above. Which of the following atrioventricular blocks is present? First-degree AV block Second-degree Mobitz type I AV block Second-degree Mobitz type II AV block Third-degree AV block

Second-degree Mobitz type I AV block The patient has a second-degree Mobitz type I AV block. A first-degree AV block refers to slowed conduction through the AV node, resulting in a prolonged PR interval, but preserved conduction of each sinus-generated impulse. In second-degree AV block, there is intermittent conduction through the AV node—some atrial impulses are conducted through to the ventricles while others are blocked. There are two types of second-degree block, Mobitz type I and type II. In second-degree Mobitz type I block, also referred to as Wenckebach, there is progressive delay in each AV conduction until AV conduction is finally blocked. On electrocardiogram, this appears as progressive lengthening of each PR interval until a QRS complex is eventually dropped and the cycle repeats. Grouped beating, or clustering of QRS complexes separated by a pause from the dropped beat, is characteristic of second degree type I AV block. Mobitz type I block is often transient, seen in association with inferior wall myocardial ischemia, medication toxicity, or cardiac surgery. Treatment is usually not necessary unless it is associated with a very slow rate with signs of hypoperfusion. In which case atropine, which increases the rate of sinus node firing and conduction through the AV node, is usually effective. The other type of second-degree AV block, Mobitz type II, is more ominous. Type II block is characterized by a constant PR interval with periodic non-conducted impulses, seen as absence of a QRS complex. Because Mobitz type II blocks are associated with progression to third-degree, or complete heart block, patients should have transcutaneous pacer pads placed in anticipation of possible need. In third-degree AV block, there is no conduction through the atrioventricular node, and an escape pacemaker is responsible for the ventricular rate. On electrocardiogram, there is no association of P waves with QRS complexes, and the ventricular rate is 40-60 beats/minute, depending on whether a junctional or ventricular escape pacemaker takes over. Pacing is usually needed in complete heart block. First-degree AV block (A) shows prolonged PR interval, but no dropped beats. Second-degree Mobitz type II AV block (C) is characterized by a constant PR interval with sporadic dropped beats. Third-degree AV block (D) is characterized by no association between P waves and QRS complexes.

A 65-year-old man presents to the ED with sudden onset of chest pain that began two hours prior to arrival. He has a history of hypertension treated with hydrochlorothiazide, hyperlipidemia treated with simvastatin, erectile dysfunction treated with sildenafil, and takes a daily aspirin. An ECG demonstrates an anterior wall myocardial infarction. Which of the patient's home medications serves as a contraindication for the use of nitroglycerin to treat this his chest pain? Aspirin Hydrochlorothiazide Sildenafil Simvastatin

Sildenafil Sildenafil is an inhibitor of cyclic guanosine monophosphate (cGMP) specific phosphodiesterase type 5. It leads to an increased release of nitric oxide resulting in smooth muscle relaxation and vasodilation. Sildenafil is a commonly used medication in men for erectile dysfunction. The combination of sildenafil and nitroglycerin can result in profound hypotension. Therefore, the use of nitrates is contraindicated in patients who also take sildenafil. The use of aspirin (A), hydrochlorothiazide (B), and simvastatin (D) are not contraindications to the use of nitroglycerin.

A 70-year-old woman with a history of hypertension presents to the Emergency Department with a complaint of abdominal pain. On physical examination, there is a pulsatile mass in the midline of her abdomen. Which of the following is the most common risk factor for the development of this condition? Diabetes mellitus Female sex Hypertension Smoking history

Smoking history Smoking is the most common preventable risk factor associated with the development of an abdominal aortic aneurysm (AAA). AAA occurs when there is localized dilation of the abdominal aorta. The risk of rupture increases as the aneurysm grows in size. A ruptured AAA has a nearly 90% mortality rate. Risk factors include smoking history, male sex, history of peripheral vascular disease, Caucasian race, and a family history of AAA. The U.S. Preventive Services Task Force recommends that men between the age of 65-75 with a history of smoking at least 100 cigarettes should be screened once for an AAA. Hypertension (C) is a less common risk factor, and the evidence for its role is still questionable. Female gender (B) is not a risk factor. Diabetes mellitus (A) is associated with a lower incidence of AAA and is protective, however, the mechanism is not well understood.

After return of spontaneous circulation from a ventricular fibrillation arrest, mortality can be determined by calculating a Cardiac Arrest Score, which takes into account which of the following factors? Left ventricular ejection fraction Pre-hospital pharmacologic treatment Systolic blood pressure Time to presentation to the emergency department

Systolic blood pressure Witnessed out-of-hospital cardiac arrest patients can be stratified by using the 3-criteria Cardiac Arrest Score developed by Thompson and McCullough. The criteria are: (1) Systolic blood pressure in the emergency department; (2) time from loss of consciousness to return of spontaneous circulation; (3) neurologic responsiveness. Criteria are SBP > 90 mm Hg (1 point; < 90 mm Hg = 0 points), time from loss of consciousness to return of spontaneous circulation < 25 minutes (1 point; > 25 minutes = 0 points) and positive neurologic responsiveness (1 point; unresponsiveness = 0 points). A score of 0 equates to 90% in-hospital mortality and 3% chance of neurologic recovery. A score of 3 equates to 18% in-hospital mortality and an 89% chance of neurologic recovery. Left ventricular ejection fraction (A), pre-hospital pharmacologic treatment (B) and time to presentation to the emergency department (D) are not criteria in the Cardiac Arrest Score. However, they do play a role in considering the overall prognosis of cardiac arrest.

Which of the following is recommended for the treatment of systolic heart failure according to New York Heart Association (NYHA) functional class? Anticoagulation for NYHA class I-IV heart failure Calcium channel blockers for NYHA class III-IV heart failure Hydralazine plus nitrates for white patients with NYHA class III-IV heart failure Spironolactone for NYHA class III-IV heart failure

Spironolactone for NYHA class III-IV heart failure Spironolactone for NYHA class III-IV heart failure is recommended for the treatment of systolic heart failure. Indications for specific medications for systolic heart failure are generally based on the patient's functional status as measured by the New York Heart Association (NYHA) functional class. Treatment with angiotensin converting enzyme inhibitors and beta-blockers are standard medical therapy for all patients with systolic heart failure regardless of functional stage. Spironolactone, in addition to standard medical therapy is indicated for more severe heart failure, specifically NYHA class III-IV. The addition of hydralazine plus nitrates to standard medical therapy is also indicated for NYHA class III-IV; however, only in African American patients. Anticoagulation (A) and calcium channel blockers (B) are not indicated in the treatment of heart failure, regardless of functional stage. Hydralazine plus nitrates for white patients with NYHA class III-IV heart failure (C) is incorrect because this combination is only recommended for black patients or those who cannot tolerate angiotensin converting enzyme inhibitors or angiotensin II receptor blockers.

A 19-year-old man presents to the ED after a syncopal event. The patient states that he was moving a large pile of wood in his backyard and then suddenly became dizzy. He awoke with his neighbor standing over him asking if he was okay. On exam, you note a loud crescendo-decrescendo systolic murmur at the left lower sternal border. He has a family history of a "heart condition." Which of the following maneuvers will accentuate the murmur? Leg elevation Squatting Standing Trendelenburg

Standing This patient has hypertrophic cardiomyopathy, which is characterized by asymmetric thickening of the left ventricular septal wall. On exam, it is associated with a loud S4 gallop and a harsh crescendo-decrescendo midsystolic murmur. Unlike the harsh murmur of aortic stenosis, which may sound similar, the murmur associated with hypertrophic cardiomyopathy does not typically radiate to the neck. Various maneuvers can help differentiate the murmurs as well. The murmur of hypertrophic cardiomyopathy is accentuated by actions that decrease blood volume in the left ventricle and increase left ventricular outflow obstruction. These actions include the Valsalva maneuver or standing. Leg elevation (A), squatting (B), and Trendelenburg (D) are all associated with an increase in left ventricular blood volume and a consequent decrease in both subvalvular outflow obstruction and intensity of the associated murmur.

What is stasis dermatitis?

Stasis dermatitis occurs with venous insufficiency and valvular incompetency. The proximal skin appears thin and brown, and may occur with distal macules, papules, red irritation, skin thickening and edema.

A 72-year-old woman presents to the ED with an acute onset of dyspnea and palpitations that began four hours prior to arrival. Vital signs include a heart rate of 144 beats per minute, blood pressure of 80/50 mm Hg, respiratory rate of 28 breaths per minute, temperature of 37.0°C, and a pulse oximetry of 88% on room air. The above 12-lead ECG is obtained. What is the most appropriate next step in the management of this patient? Anticoagulation with enoxaparin followed by warfarin Chemical cardioversion using procainamide Rate control using diltiazem Rate control using esmolol Synchronized cardioversion

Synchronized cardioversion This patient has atrial fibrillation with a rapid ventricular rate and is hemodynamically unstable (BP 80/50 mm Hg and pulse ox 88%). This dysrhythmia needs to be emergently corrected in order to stabilize the patient; failure to do so could result in sudden cardiac death. In such circumstances, emergent synchronized cardioversion is required. If the patient is hemodynamically stable, the approach to management (rate vs. rhythm control) depends on the time of dysrhythmia onset. If the onset is unknown or greater than 48 hours, then cardioversion should be delayed until the patient can be adequately anticoagulated with enoxaparin (A) followed by warfarin. In the interim, rate control would be accomplished with either esmolol (D), a short acting ß-blocker, or diltiazem (C), a calcium channel blocker. For patients with stable paroxysmal atrial fibrillation and duration of onset < 48 hours, chemical cardioversion using procainamide (B) can be attempted. In unstable patients, regardless of the rhythm duration, synchronized electrical cardioversion is recommended. Alternatives to procainamide include amiodarone, ibutilide, and, to a lesser degree, flecainide.

A man presents to the ED with angina during exertion that has increased in intensity and duration over the past 18 hours. An ECG shows T wave inversion in 4 leads. Which of the following risk stratification tools is used in the acute management of this type of angina? CHADS-2 Score Framingham 10-year Cardiovascular Disease Score Reynolds Score TIMI Score

TIMI Score Nearly 5 million Americans present to the emergency department with angina, with about 1.4 million of these patients getting admitted under the diagnosis of unstable angina (UA) or non ST elevation myocardial infarction (NSTEMI). Of the acute coronary syndromes, unstable angina (UA) and non ST-segment elevation myocardial infarction (NSTEMI) are similar conditions of subtotal coronary occlusion with myocardial ischemia or infarct, but both are distinctly different than ST-segment elevation myocardial infarction (STEMI) which represents complete coronary occlusion and myocardial infarct. UA is characterized by anginal episodes at rest or with minimal exertion that increase in intensity or frequency with time and are often accompanied by dynamic ECG changes. The initial evaluation of UA and NSTEMI includes risk stratification to predict the occurrence of adverse outcomes. One such guideline is the Thrombolysis in Myocardial Infarction (TIMI) risk tool. This seven point scoring tool utilizes historical facts, ECG findings and laboratory results to guide further therapeutic options. A higher score places the patient into a more invasive paradigm that usually includes cardiac catheterization, whereas a lower score favors a more conservative approach with medication options. The CHADS2 Score (A) is used to predict the likelihood of stroke in patients with atrial fibrillation, the results of which guide anticoagulant and antiplatelet therapy. The Framingham 10-year CVD (B) Score is a tool used to determine the likelihood of a patient having a heart attack in the next 10 years. It is not used in the acute evaluation of UA or NSTEMI. The Reynolds Score (C) is a tool used to predict a 10-year risk of MI, stroke or major heart disease. It too is not used in the acute evaluation of UA.

Which of the following describes ECG findings in Wolff-Parkinson-White Syndrome? The delta wave reflects rapid conduction through the AV node The PR interval is shortened in sinus rhythm The QRS complex is typically narrow in sinus The QT segment is prolonged in sinus rhythm

The PR interval is shortened in sinus rhythm Wolff-Parkinson-White (WPW) syndrome is a ventricular preexcitation syndrome that involves an anatomic accessory pathway that bypasses the AV node and forms a direct electrical connection between the atria and ventricles. Patients with this accessory pathway are at high risk of supraventricular tachydysrhythmias. The classic triad of WPW syndrome can be seen when the patients are in sinus rhythm: (1) a shortened PR interval (2) a fused QRS complex and (3) a delta wave at the beginning of the QRS complex. These classic signs are typically not seen when the patient is in a tachydysrhythmia. The shortened PR interval is due to rapid conduction of the atrial impulse through the accessory tract, which is able to conduct the impulse faster than the physiologic pace of the AV node. The fused, widened QRS complex results from activation of the ventricles by both the physiologic AV node and the aberrant impulse from the accessory tract. The initial part of this widened complex, called the delta wave, represents ventricular activation by the impulse conducted through the accessory pathway while the remainder of the QRS complex represents normal ventricular depolarization from the impulse travelling through the AV node. The classic delta wave of WPW represents conduction through the accessory tract, not the AV node (A). The QRS complex (C) is typically fused or widened due to ventricular activation by both the accessory pathway and the AV node. The QT interval (D) is typically unchanged in WPW syndrome.

Angina may be associated with which psychological disorders?

The anxiety disorders, such as panic disorder and generalized anxiety disorder.

A 63-year-old man is brought to the ED by paramedics secondary to aphasia that began 30 minutes prior to arrival. The man is unable to give any history and has never been to your hospital. His vital signs are T 37°C, BP 180/98, HR 90, and RR 20. His exam is unremarkable except for profound expressive aphasia. Capillary blood glucose is 124 mg/dL. His ECG shows normal sinus rhythm with normal ST segments and T waves. CT scan of the head is normal. Sixty minutes after arriving to the ED, the patient's aphasia resolves. Which of the following are the most appropriate diagnosis, treatment, and disposition for this patient? Hypertensive emergency; start IV antihypertensive therapy and admit to the medical intensive care unit Stroke; administer tissue plasminogen activator; admit to stroke unit Transient ischemic attack; administer aspirin and dipyridamole; admit to stroke unit Transient ischemic attack; administer low molecular weight heparin and warfarin; discharge home with close neurology follow up Transient ischemic attack; start heparin infusion; admit to stroke unit

Transient ischemic attack; administer aspirin and dipyridamole; admit to stroke unit The 2009 AHA/ASA guidelines define transient ischemic attack (TIA) as "a transient episode of neurological dysfunction caused by focal brain, spinal cord, or retinal ischemia without acute infarction." Previously, TIAs were differentiated from strokes by the duration of symptoms—with TIAs lasting less than 24 hours. However, it has since become evident from imaging studies that many patients with symptoms lasting less than 24 hours have had a stroke. The latest class IA treatment recommendation is for aspirin alone or aspirin plus dipyridamole. Patients diagnosed with a TIA should be admitted for assessment of stroke risk factors because the 90-day risk of subsequent stroke is ~ 9.5%. The risk of stroke increases in patients older than 60 years, with a history of hypertension or diabetes mellitus, and whose symptoms last > 10 minutes or are associated with weakness or speech impairments. Even though hypertension is a risk factor for TIA (A) or ischemic stroke, and can cause cerebral impairment, the AHA/ASA recommends permissive hypertension if the patient is not receiving thrombolytics. In such cases, blood pressure should be acutely lowered only if it remains persistently > 220/120 mm Hg with a goal of reducing by 15% within 24 hours. This patient's symptoms have resolved (B), so he is not a candidate for thrombolysis. Unfractionated and low molecular weight heparin (E) have not been shown to improve outcomes in stroke patients and increase the risk for intracerebral hemorrhage. Warfarin (D) is typically prescribed to patients with atrial fibrillation to reduce the risk of stroke from a thrombotic embolism. It should not be started until a full stroke workup has been performed.

How many grams of sodium can one eat per day on a low sodium diet?

Under 2 grams.

What is the most common cause for syncope in a patient with a non-specific history, normal physical exam and normal EKG? Dysrhythmia Hypovolemia Unknown Vasovagal

Unknown Syncope is defined as a sudden temporary loss of consciousness with the inability to maintain postural tone. It is a symptom with a wide variety of life-threatening causes. Risk factors for syncope include cerebrovascular disease, cardiac manifestations, and hypertension. Despite extensive workups, no cause for syncope is found in approximately 50% of cases. When a diagnosis is made, most causes of syncope are benign and have favorable outcomes. Patients with preexisting cardiovascular disease and syncope from any cause are at the greatest short- and long-term risk of mortality. Syncope from cardiovascular causes is associated with increased mortality, whereas syncope due to neurocardiogenic, orthostatic, and medication-related syncope is not associated with increased mortality. Cardiac dysrhythmias (A) are the most common cause of cardiac syncope. Brady dysrhythmias such as heart block can lead to syncope in addition to tachydysrhythmias such as ventricular tachycardia and supraventricular tachycardia. Orthostatic hypotension, dehydration, and hypovolemia (B) can cause syncope. The most susceptible individuals are elderly patients taking blood pressure medications or diuretics. Syncope can also be the presenting symptom secondary to hemorrhage such as seen with an ectopic pregnancy. Vasovagal (D), or situational syncope, is a common cause of syncope. Multiple factors contribute to these attacks. There is typically a relatively low blood volume. Various circumstances can then lead to the syncopal event. If there is underlying fear or anxiety, the vasomotor center increases heart rate in order to increase cardiac output. However, due to a relatively low blood volume, the heart is unable to meet the requirement. The high sympathetic activity is always modulated by vagal outflow, in these cases leading to excessive slowing of heart rate. This leads to a paradoxical vagal response that causes syncope. The tilt-table test typically evokes the attack.

A 55-year-old man with no history of atherosclerotic cardiovascular disease presents to your office with questions about his cholesterol. He was previously taking atorvastatin, but stopped because his fasting low-density lipoprotein (LDL) level decreased below 100 and he was following the lifestyle modifications you previously recommended. Based on the 2013 American College of Cardiology/American Heart Association Cholesterol Guidelines, which of the following is the most appropriate next step in his management? Encourage the patient to continue with lifestyle modifications only Restart the atorvastatin at the previously prescribed dose Start the patient on a different statin and screen for diabetes Use a cardiovascular risk calculator to determine the patient's 10-year risk for atherosclerotic cardiovascular disease

Use a cardiovascular risk calculator to determine the patient's 10-year risk for atherosclerotic cardiovascular disease The American College of Cardiology and the American Heart Association (ACC/AHA) released a new set of guidelines in November 2013 with information about treating blood cholesterol as a means of reducing the risk of atherosclerotic cardiovascular disease in adults. Four groups were identified as benefiting from initiation of statin therapy: patients with atherosclerotic cardiovascular disease, patients with LDL levels ≥190 mg/dL, patients aged 40-75 with diabetes and an LDL level of 70-189 mg/dL, and patients with LDL level of 70-189 mg/dL and a 10-year atherosclerotic cardiovascular disease risk ≥7.5%. The panel recommends calculating the 10-year risk for atherosclerotic cardiovascular disease as a way of initiating the conversation about prevention with patients. The new recommendations no longer promote the "treat to goal" strategy that the previous ATP III guidelines used. Lifestyle modifications (A) are the starting point of all efforts for medical providers in preventing atherosclerotic cardiovascular disease in patients, but are not the only tool. Atorvastatin (B) may be indicated, but only after determining the patient's 10-year risk for atherosclerotic cardiovascular disease, since he does not fall into the other three groups that benefit from statin therapy. Choice of a statin (C) is based on the need for high, medium or low-intensity therapy and the new guidelines provide recommendations for which statins to use at each level of treatment needed. Based on the patient's 10-year risk for atherosclerotic cardiovascular disease and fasting lipid levels, the medical provider may choose to continue with atorvastatin or use a different agent. Screening for diabetes is not specifically discussed in the new guidelines although a diagnosis of diabetes in patients 40-75 years old with an LDL level of 70-189 mg/dL is considered an indication for statin therapy.

An obese 45-year-old man works for the local electric company as a utility-pole lineman. He presents with painful, tortuous superficial veins on his lower legs. These areas are not associated with erythema, warmth or skin breaks. Which of the following initial plans is most appropriate at this time? Dangle your legs at night over the side of your bed for 30 minutes Increase your daily exercise routine Schedule an appointment with a vascular surgeon Use compressive stockings during the day

Use compressive stockings during the day Venous insufficiency is mainly a chronic condition in which the "musculovenous pump" of the distal extremities does not properly return deoxygenated blood the heart, resulting in retrograde flow and distal blood back-up. A major cause of this is venous valvular incompetence, in which the vein leaflets do not operate properly. There are many causes of valvular incompetence, the most common of which is repeated vein injury, which can occur in utility pole lineman, or prior deep vein thrombosis. Two complications of valvular incompetence are varicose veins (with or without symptoms) and thrombophlebitis (inflamed superficial and/or deep vein inflammation due to a blood clot). Varicose veins are enlarged and twisted veins which have been dilated due to incompetent valves, retrograde blood flow and back-up of venous blood. Varicose veins most commonly occur in the superficial veins of the legs, especially when standing. They can be asymptomatic but have unsightly cosmesis, or they can be painful and be associated with distal edema, venous eczema, lipodermatosclerosis (skin thickening) and ulceration. They are more common in women, and are associated with pregnancy, obesity, aging, menopause, abdominal straining and prolonged standing. A complication is progression into thrombophlebitis. Treatment includes leg elevation, graduated compression stockings with or without intermittent pneumatic compression, aspirin or NSAIDs and topical anti-inflammatory gels. Leg elevation (A), especially at night, is recommended. Although somewhat counterintuitive, there is no supporting evidence to recommend regular exercise (B) as a viable treatment of symptomatic varicose veins. There are several surgical methods used in treating symptomatic varicose veins. However, this patient has not tried any conservative measures yet, and as such, referral to a surgeon (C) is not recommended at this time.

An 18-year-old male undergoes an electrocardiogram as part of a sports physical for his college soccer team. The electrocardiogram was read as abnormal, and he was sent to the ED for evaluation. His electrocardiogram is shown above. Which of the following is the most common complication found with the suspected diagnosis? Atrial fibrillation Eisenmenger's syndrome Type 2 second-degree heart block Ventricular fibrillation

Ventricular fibrillation The patient's ECG is suggestive of Brugada syndrome. Brugada syndrome is a hereditary condition that leads to abnormal myocardial depolarization and predisposes otherwise young healthy individuals to ventricular fibrillation and sudden cardiac death. Several variants of Brugada syndrome exist, which involve mutations in the genes responsible for transmembrane sodium, calcium, and potassium ion channels in the heart. The incidence of Brugada syndrome is highest in Asian populations and is lower in groups originating from Western Europe. Most patients with Brugada syndrome are asymptomatic, and the disorder is identified incidentally via an abnormal ECG. Palpitations, near-syncope, and syncope may be reported. There may also be a family history of sudden cardiac death. Classic ECG findings of Type 1 Brugada syndrome include coved ST segment elevation followed by an inverted T wave in leads V1 to V3. If Brugada syndrome is suspected based on ECG findings or clinical history, patients should be referred to cardiology for further testing to rule out structural heart problems and confirm the diagnosis of Brugada syndrome. Medications with sodium channel blockade, such as flecainide and procainamide, should be avoided. The definitive treatment for Brugada syndrome is placement of an implantable cardioverter defibrillator device. Atrial fibrillation (A) occurs in 10 - 20 % of patients with Brugada syndrome and is associated with higher risk for ventricular tachyarrhythmias and disease severity. Eisenmenger's syndrome (B) occurs when a left-to-right cardiac shunt, such as from a ventricular septal defect, switches direction to become a right-to-left cardiac shunt. Type 2 second-degree heart block (C) is not associated with Brugada syndrome.

Capture beats and fusion beats confirm the diagnosis of which cardiac dysrhythmia? Atrial fibrillation Supraventricular tachycardia Ventricular fibrillation Ventricular tachycardia

Ventricular tachycardia Ventricular tachycardia can be diagnosed when capture beats or fusion beats are seen in the setting of a wide complex tachycardia. This is especially helpful when trying to distinguish between ventricular tachycardia and supraventricular tachycardia with aberrant conduction. Supraventricular tachycardia with aberrancy can produce a wide complex tachycardia that mimics ventricular tachycardia. It is important to distinguish between the two because ventricular tachycardia is a more serious condition and treatment is different. During ventricular tachycardia, there is AV dissociation in which independent pacing of the atria and ventricles occurs. The SA node still paces the atria, but the larger ventricular complexes often hide the P waves. However, P waves can be seen occasionally. When a sinus paced depolarization is able to conduct to the ventricles, it can produce a normal-appearing QRS; this is called a capture beat. More commonly, an atrial depolarization is able to conduct only partially before it encounters depolarization coming from the ventricles; this produces a fusion beat, which is a blending of a normal QRS with a PVC like complex. Capture beats and fusion beats do not occur during supraventricular tachycardia, making their presence useful in differentiating the two. Capture and fusion beats are not seen in atrial fibrillation (A), supraventricular tachycardia (B), or ventricular fibrillation (C).

What is the most common ECG abnormality seen in acute pericarditis?

Widespread, concave-up ST-segment elevation with PR depression.


संबंधित स्टडी सेट्स

Complementary& Alternative Medicine (CAM)

View Set

Repaso de Pobre Ana Bailó Tango (Cap. 1-2)

View Set

Learning plan 2 homework assignment

View Set

Fundamentals Practice Exam (HESI)

View Set

Chapter 72: Care of Pt with Male Reproductive Problems EAQ

View Set

Capítulo 15: Los avances tecnológicos

View Set

Real Estate Valuation and Market Analysis

View Set

Why was the Missouri Compromise necessary, what agreements did it include, and what were the consequences of the compromise on the nation?

View Set

Patho final exam (Chapters 30,32,33,37,39,40,41,46 and comprehensive)

View Set